Conference Notes 10-11-2017

Denk/Traylor   STEMI Confernce

Inferior Posterior MI with RV involvement. Note the ST elevation greater in Lead 3 than Lead 2.

Inferior Posterior MI with RV involvement. Note the ST elevation greater in Lead 3 than Lead 2.

Patient with an Inferior/Posterior/RV MI.  With hypotension.  Cardiology advised initial 1 liter of saline.  If that does not solve hypotension problem, next start a pressor.  No difference in outcomes between dopamine and norepi.  Dopamine has been shown to have more arrhythmia.  Harwood comment: If the patient is bradycardic, dopamine has more chronotropy and is preferred. because you will raise the heart rate. 

Editors comment: If the patient is not bradycardic, my reading of the below reference suggests that norepi may be the preferred choice.

 

New England JournalReference: The trial included 1679 patients with shock of all causes, of whom 858 were assigned to dopamine and 821 to norepinephrine. The baseline characteristics of the groups were similar. There was no significant between-group difference in the rate of death at 28 days (52.5% in the dopamine group and 48.5% in the norepinephrine group; odds ratio with dopamine, 1.17; 95% confidence interval, 0.97 to 1.42; P=0.10). However, there were more arrhythmic events among the patients treated with dopamine than among those treated with norepinephrine (207 events [24.1%] vs. 102 events [12.4%], P<0.001). A subgroup analysis showed that dopamine, as compared with norepinephrine, was associated with an increased rate of death at 28 days among the 280 patients with cardiogenic shock but not among the 1044 patients with septic shock or the 263 with hypovolemic shock (P=0.03 for cardiogenic shock, P=0.19 for septic shock, and P=0.84 for hypovolemic shock, in Kaplan–Meier analyses).  De Backer N Engl J Med 2010; 362:779-789

Balloon pump therapyhas not shown a mortality benefit in cardiogenic shock but all the cardiologists present felt that ballon pumps are useful in supporting the patient in the short term.  They usually use it in patients with high pulmonary capillary wedge pressure or patients who need improved coronary flow.  

Editor's note:  This feeling generally corresponds to a brief lit search on this topic.  No overall mortality benefit but some patients did have hemodynamic improvement with balloon pump therapy.

Transvenous pacers are more easily placed in the cath lab than in the ED because of flouro.  Only place a transvenous pacer in the ED if the patient is unstable due to bradycardia or heart block. 

Factors making angioplasty more risky:  If a patient has prior CABG it makes angioplasty much more difficult. Patients who have had cardiac arrest prior to going to the cath lab have worse outcomes overall.  When in a difficult decision making situation regarding whether or not to take a patient to the cath lab, be sure to document the collaborative decision making that you had with the cardiologist.   Lovell comment: Your mantra should be "shared decision-making"

Polycythemia Vera can result in sluggish coronary flow and MI.

Regan    Disaster Planning

The new paradigm after the Columbine event is to extricate victims as soon as possible.  Most SWAT teams have tactical medics or doctors who go in with SWAT team to get shooting victims out as fast as possible and get them to medical care. This change has improved survival.

It is common for people to "freeze" and not react properly to a terrorist or mass shooting event.  There are clear reports of this happening during the 9-11 event.  First responders will need to give victims clear direction to get them to safety.

When EMS contacts the ED, key pieces of information include: are there pediatric patients involved, is there a HAZMAT component to this event, the number of victims, and the estimated time of arrival of patients to the ED.

If there is a disaster or terrorist or shooting event that affects our hospital a CODE TRIAGE will be called.   The Emergency Operations Center (EOC) will be in the Conference Center Auditorium. Hospital Administrative personnel will run the EOC.  We as ED physicians will provide triage and patient care.   In a disaster the goal of patient care is not to provide "standard of care", but rather "sufficient care."  Basically that means you provide basic stabilization care to patients, not definitive care.

"The term “altered standards” has not been defined, but generally is assumed to mean a shift to providing care and allocating scarce equipment, supplies, and personnel in a way that saves the largest number of lives in contrast to the traditional focus on saving individuals."  AHRQ Publication No. 05-0043April 2005

In the event of a mass casualty incident, call the Medical Director of Disaster Medicine (Liz or Sean). 

We then will huddle with ED nursing and physcian staff to assign duties.

There is a disaster box in the back charting room.  It has written role cards for staff.   Doctors will assigned to Triage (1 physician and Charge nurse in the ambulance bay)  or to Treatment of patients. Both attendings and senior residents will be treating injured patients.   Junior residents will take on the role of moving our patients already in the ED out of the ED to make room for the injured patients.

Physicians report to the Charge MD who will be Sean, Liz or the most senior ED physician in the ED.

The Trauma Surgeons will be in the OR's.

Triage will occur in the ambulance bay.  Treating physicians in the ED will stabilize patients and identify which patients require the OR as soon as possible.

Use IO access preferentially in a disaster situation.  DO NOT spend the time to place a central line or ultrasound guided line.

Employ your Trauma skills to stabilize patients: assess/protect airway, use needle thoracostomy/chest tubes to treat pneumothorax and hemothorax, give blood, use TXA liberally,  control extremity bleeding with tourniquets, get patients to the OR.

Additional physicians who are called in should park in garages A,B, or C.  When you get to the hospital report to the ED Administrative Offices.

Do not self-dispatch to the hospital in the event of a mass casualty incident.  We don't want too many people here at one time.  Only come in if you have been contacted to come in.

EM1's      Pecha Kucha

Chinwala       Sleep for EM Physicians

Unfortunately I missed this excellent lecture

Jurkovic   Blessed

Heart warming lecture describing how senior residents have helped the interns learn and battle through adversity.

Pastore         Priapism

Basic management approach to low flow priapism

Basic management approach to low flow priapism

Blood gas findings indicating low flow (ischemic) priapism

Blood gas findings indicating low flow (ischemic) priapism

Low flow priapism is compartment syndrome of the penis.   Give IV pain medication and perform a dorsal penile block.  Aspirate blood from the corpus cavernosum.  The blood will look like motor oil.  Irrigate with saline.  Inject phenylepherine .   After detumesence apply an elastic bandage on the penis.

For priapism associated with sickle cell disease consult hematology for an exchange transfusion.

Kentor    HINTS Exam

Used in patients with continuous vertigo for more than 24 hours. Is more reliable identifying a central cause of vertigo than MRI.

With rapid head turning, if eyes can stay on target then it may be central vertigo.    With regard to nystagmus if has direction changing nystagmus or vertigal nystagmus that suggests central cause.

Snip20171015_7.png

Kishi    U/S Guided Forearm Nerve Blocks

Patrick described the procedure of using soft tissue ultrasound to identify and aid local injection around the median, radial, and ulnar nerves in the forearm .

Ohl     Subarachnoid Hemorrhage

CT is close to 100% sensitive in the first 6 hours.  Beyond 6 hours in the setting of a negative CT head, the next study would be LP.

Treat BP with Nicardipine.  Keep the patient's head elevated at 30 degrees.

Vasospasm is more common in younger patients, smokers, larger bleeds, and patients with HTN.

 

Conference Notes 9-27-2017

Barounis     Ventilator Management

You will use Assist control/Volume control ventilator mode 99.9% of the time in the ED. There is rarely a need to use another mode in the acutely ill ED patient. 

5 Steps to setting the vent

1. Select Assist Control/Volume Control

2. Tidal volume.  Set your tidal volume to keep plateau pressure at or below 30.  Higher pressures cause lung injury. Low tidal volumes are lung protective.  Number Needed to Treat using low tidal volumes to save 1 life is 11!  Start at 6 ml/kg.  Distilling this down to a real basic guideline for ER docs is give 500 ml for guys and 400ml for women.  You can adjust up and down +/- 50ml if the patient is particularly tall or short.

3. Pick your RR.  If the patient has bronchospasm, you need to use a low rate (10 breaths/min) to give the patient more expiatory time so you are not causing breath stacking.  So, for asthma 10 breaths/min,  for hypoxia or to protect airway 20 breaths/min, and for severe metabolic acidosis 30 breaths/min.   Logan Traylor comment: For respiratory acidosis due to asthma or COPD, patients still need a low rate to avoid breath stacking. As the clinician, you will need to accept some respiratory acidosis.    Dave agreed and added furtehr comment: There is some data that patients who are acidotic after cardiac arrest may do better with a higher PCO2 level and an acidotic ph than ventilating them rapidly and lowering the PCO2 and causing cerebral vasoconstriction. 

4. Set PEEP.  You want to keep the driving pressure (plateau pressure-peep) less than 15.   Start with a peep at 8 for the average overweight ED patient.  After 20 minutes, do an inspiratory hold and figure out the driving pressure and increase the peep as needed to lower the driving pressure to 15 or less.

5. Set the FIO2.  Base your FIO2 on the O2 sat.  Try to keep the O2 sat around 95%.  Avoid hyperoxia and hypoxia.

 

If peak pressure is high and plateau pressure is OK then you have a resistance problem.  Resistance issues include: patient biting the tube, mucous plug, blood in the ET tube, tension pneumothorax, and asthma.

Nand   The 2 Midnight Rule

Patient who are inpatients for more than 2 midnights after admission are presumed to be appropriate for Medicare Part A payments.  Our documentation needs to reflect the necessity of that admission.

The clock starts when the patient begins to receive care in the ED. If the patient is in the ED at midnight, that counts as the first midnight.

One exception is a patient placed on a ventilator.  They can all be made inpatients even if they will stay less than 2 midnights.

Keeping patients for social or safety reasons who have no other medical issues should in general be OBS stays.  If the patient has some concurrent medical issue they may be appropriate for admission.

Patients with symptom-based diagnoses (chest pain, abdominal pain) should be OBS.  If you think patient will go home the next day, make them an OBS.

Don't write" we will admit to OBS"  Medicare will not approve this type of explanation. We have to avoid using both the terms Admit and OBS in the same chart.  When both terms (Admit, OBS) are used in the same chart Medicare uses that lack of clarity to decline payment.   Better to write "we will place patient in OBS for further evaluation"  or " we are admitting patient for management of pancreatitis"

If you need to change a bed request. 

1. Cancel the initial level of care order.  2. Cancel the initial bed request.  3. Place the new bed request.   If you don't do this in the correct order medicare will deny the admission.   

Mounica Donapudi comment: 1. Right click the level of care and click cancel d/c.  2. Rght click the the bed request and click reorder. 3. change the bed request order and sign.

Tran    Human Trafficing

Unfortunately I missed this excellent lecture.

Traylor   Admin Update

You need to write the sepsis re-eval (..sepsis macro) for any patient with a lactate over 4 or a MAP<65.    Give 30ml/kg of LR or saline.  3 liters should work for most patients.  You have to write the reassessment note within an hour after the IV fluid bolus.   If you have concerns about volume overload, document that concern and order >125ml/hr (126ml/hour is acceptable)

April Kennedy    Dental Emergencies

Snip20170927_1.png

How to describe teeth.

Snip20170927_2.png

Cross sectional anatomy of a tooth.

Pulpitis is the most common cause of tooth pain.&nbsp; Treat with pain control and antibiotics.&nbsp; If there is an open area on the crown, it may help to place a covering on the tooth with Temrex. Patientswill need dental referral for extraction o…

Pulpitis is the most common cause of tooth pain.  Treat with pain control and antibiotics.  If there is an open area on the crown, it may help to place a covering on the tooth with Temrex. Patientswill need dental referral for extraction or root canal.

Patients will need tooth extraction as definitive management of pericoronitis. In the ED start them on antibiotics and pain meds and refer them to their dentist or an oral surgeon.

Patients will need tooth extraction as definitive management of pericoronitis. In the ED start them on antibiotics and pain meds and refer them to their dentist or an oral surgeon.

Dry socket occurs when the blood clot that has developed in the socket since dental extraction becomes dislodged from the socket. This leaves the nerve exposed and is quite painful.&nbsp; Dry socket is more common in smokers, patients over age 25, p…

Dry socket occurs when the blood clot that has developed in the socket since dental extraction becomes dislodged from the socket. This leaves the nerve exposed and is quite painful.  Dry socket is more common in smokers, patients over age 25, patients with poor oral hygeine and patients who use straws to drink liquids.

Academic Life in EM: Trick of the Trade: Extra-Oral Reduction Technique for Anterior Mandible Dislocation

  1. Place the patient in either sitting or supine position.
  2. The provider should stand in front of the patient.
  3. The provider places their thumb on the patient’s cheek, on the mandibular ramus and coronoid process of the dislocated mandible, and applies persistent pressure posteriorly (figure 3).
  4. The fingers are placed behind the angle of the mandible to stabilize the grip.
  5. At the same time on the opposite side, the provider places their fingers from the other hand on the angle of the mandible and pulls, applying anterior force (figure 4). Note that this maneuver causes further anterior dislocation of the ipsilateral TMJ, rotates the jaw, and facilitates contralateral TMJ reduction.
Snip20170927_8.png

Once one side of the dislocation is reduced, the other side will usually go back spontaneously. If that doesn’t work, repeating the same maneuver with minimal force will usually result in success. Also consider applying posterior force on both coronoid processes at the same time if the above strategy doesn’t work.

If a primary tooth (baby tooth) is avulsed do not replace it.  If a permanant tooth is avulsed, rinse the tooth with saline. Save the tooth in tooth saver liquid, the patients own saliva, or milk.  Replace the tooth and splint it in place.  Give doxycyline as antibiotic prophylaxis to adults. Update their tetanus shot.  Patient will need dental or oral surgery referral for a root canal.   Replacing the tooth actually supports the alveolar bone and optimizes the bone for eventual dental prosthetic if needed.  Dr. Kennedy advises replacement of the tooth up to 24 hours out.  You should advise patients that they will likely loose the tooth and need a prosthetic.

You can cover Ellis fractures with Calcium hydroxide or Dermabond.

You can cover Ellis fractures with Calcium hydroxide or Dermabond.

 

Dental Lab

 

Conference Notes 9-6-2017

ACMC EM Conference Notes(filling in for Dr. G)

Editor's note:  Much Thanks to Dr. Lovell for writing these notes when I missed Conference on 9-6-2017.

8 am:  Oral Boards:  Dr. Williamson and Dr. Okubanjo

--Thyroid storm:  Consider in setting of hyperthyroidism, fever, AMS, sympathetic surge, and always look for precipitating event (eg infection, CNS, cardiac event).

Management:

  • IVF + glucose
  • acetaminophen (no NSAIDS or asprin -> displaces thyroxine from proteins)
  • propranolol to dampen sympathetic surge, also blocks T4 to T3
  • hydrocortisone or dexamethasone (shield from adrenal insufficiency + decrease peripheral conversion)
  • Thionamides:  methimazole or propylthiouracil (block new production)
  • after blockade by thionamide, wait at least 60 minutes then give sodium iodide or potassium iodide (SSKI) or Lugol’s iodine to block thyroid hormone release

 

--Acute Angle Closure Glaucoma:  think about it in patient with a headache, vomiting.  Eye symptoms may be initially more subtle.  “Mid-dilated fixed pupil” for the boards.

Snip20170924_5.png

 

Beta blocker and acetazolamide and alpha agonist to decrease aqueous humor production.  Topical steroids as anti-inflammatory agent.  May use oral glycerol instead of IV mannitol (both osmotic agents) if no diabetes and able to take po. Treating pain/nausea may also help decrease IOP. Pilocarpine administration should be delayed an hour, as initial elevated IOP can cause temporary ischemic iris paralysis.  Initial agents are given time to work, then pilocarpine helps with constricting ciliary muscle and relieving pupillary block.  Laser peripheral iridotomy is definitive treatment and can be coordinated with ophthalmology. 

 

 

--“Fight Bite” from clenched fist injury, concern for infected joint, needs IV antibiotics, usually Unasyn if no PCN allergy, doxy an alternative.  Buzzword is Eikenella but usually polymicrobial.  Avoid first generation cephalosporins.  Update tetanus, xray to eval for fracture/foreign body.  May need washout in OR.

Snip20170924_6.png

 

Snip20170924_7.png

9:00 am Dr. Kyle Bernard:  M&M.  Multiple excellent cases/learning points-a few pearls from 2 cases:

Cardiogenic shock:  sometime the subtle shock.  Patient with advanced CHF +/- ACS +/- dysrhythmia +/- valve disease +/- cardiac drug toxicity may be obviously short of breath, clammy and hypotensive, but they may also be living a very fine balance and initially present looking ok, maybe a little tachycardic, but with normal BP.  Will need to treat with O2, pressors, inotropes, possible IABP.  Key is recognition and early consultation with cardiology.

Important concept discussed-Lactate as reflecting catecholamine surge rather than anaerobic metabolism.  May see elevated lactate with compensated or “occult” shock in a patient with initially normal blood pressure who then crashes.

If you want to geek out on lactate, listen to contrarian intensivist Paul Marik:

https://intensivecarenetwork.com/understanding-lactate-paul-marik/

then read some retorts:

https://emcrit.org/emcrit/smacc-back-marik-lactate/

 

Heimlich valve in spontaneous pneumothorax:  we have a “pneumothorax kit” and can also find a “pigtail catheter kit.”  Both insert a small catheter and Heimlich valve. The pneumothorax kit uses a catheter over a needle attached to a syringe, and the pearl is that if you remove the syringe while the needle/catheter are in the chest, the pneumothorax may decompress and you’ll be unable to thread the catheter.  The pigtail catheter uses a wire/seldinger technique-more steps, but familiar to emergency physicians.

“Pigtail catheter” (wire, seldinger technique)

Snip20170924_8.png

 

Snip20170924_9.png

“Pneumothorax kit”-catheter over needle

 

 

10:00 am Dr. Liz Regan:  “Thinking outside the box-unusual uses for medications”

TXA:  mechanism of action: Tranexamic acid is a synthetic analog of the amino acid lysine. It acts as an antifibrinolytic by reversibly binding lysine receptor sites on plasminogen or plasmin.

Indications: Hemorrhagic Shock, Hyphema, post partum bleeding, menorrhagia, epistaxis, gum bleeding, hemoptysis. 

 

Ketamine:  NMDA receptor antagonist, opioid receptor agonist.

Uses include:  procedural sedation, analgesia, medical restraint, post intubation sedation, adjunct to asthma/COPD treatment (bronchodilator), anxiolysis/eg facilitate BIPAP

 

Magnesium: 

Uses:  Torsades, Magnesium deficiency, asthma/bronchospasm, pre-eclampsia/eclampsia, Afib RVR, Tocolytic, SAH induced vasospasm, migraine, constipation

 

Haldol:  Antipsychotic, dopamine antagonist, serotonin agonist

Acute Psychosis, chemical restraint, cyclic vomiting, gastroparesis, hypemesis cannabinoid syndrome, migraines

 

11:00 am -12:30 pm:  Head and Neck Trauma, Drs. Maddelynn Hawkins, Amanda Friend and Graeme Twanow.  Unfortunately I missed this outstanding lecture.

 

Conference Notes 9-20-2017

Cirone/DeStefani     Oral Boards

Case 1.   20 yo female who appears intoxicated. Patient was brought in by her roommate who was concerned about her.  Patient was at dorm party the night before.  On exam patient is disoriented.  She is tachycardic and her mucosa is dry. ABG shows metabolic acidosis.  Patient also has an osmolal gap.  Patient was treated with IV fomepizole.   Toxic alcohol screening was positive for ethylene glycol.

Anion and Osmolar Gap Calculations.&nbsp; For the calculated osmolality, you can also factor in ETOH by adding ETOH/4.6.&nbsp; The way to remember 4.6 is that there are 4 six packs in a case of beer. For easy calculations on boards round 4.6 to 5.

Anion and Osmolar Gap Calculations.  For the calculated osmolality, you can also factor in ETOH by adding ETOH/4.6.  The way to remember 4.6 is that there are 4 six packs in a case of beer. For easy calculations on boards round 4.6 to 5.

Snip20170923_2.png

 

Case 2. 19yo female presents with "flu symptoms" with fever and muscle aches.  Patient had some nausea and vomiting.  She also had a bloody nose.  She is tachycardic and hypotensive.   On exam patient has a petechial rash. Labs show thrombocytopenia and AKI.  Lactate is also elevated.   Diagnosis is disseminated meningococcemia with DIC.

Meningococcemia Rash

Meningococcemia Rash

It can be tough to differentiate these diagnoses but DIC has coagulopathy with prolonged PT/INR whereas the others do not.

It can be tough to differentiate these diagnoses but DIC has coagulopathy with prolonged PT/INR whereas the others do not.

Snip20170920_4.png

 

Case 3.  19 yo female with blunt trauma to right eye.  Patient has worsening visual acuity of injured eye with associated afferent pupillary defect.   CT shows retrobulbar hematoma.   Treatment is lateral canthotomy.

Snip20170920_6.png

Felder      OB-Gyne Jeopardy

Retained placental tissue can result in persistent vaginal bleeding days after delivery.

If a patient has a seizure in the second half of pregnancy or in the post-partum period, treat with magnesium for ecclampsia.  Mag dosing is 6g IV followed by a 2g/hr drip.

Snip20170920_7.png

Molar pregnancy

Snip20170920_9.png
Management approach to gestation trophoblastic disease.&nbsp; It is a bit beyond what is needed to be known for EM but it may be helpful to know that OB will follow up with serial HCG's and a CXR and base further management on those screening tests.

Management approach to gestation trophoblastic disease.  It is a bit beyond what is needed to be known for EM but it may be helpful to know that OB will follow up with serial HCG's and a CXR and base further management on those screening tests.

If you have some clinical suspicion for cervicitis or PID, just treat them in the ED.  Don't wait for the results of testing.  Many patients cannot be contacted after they leave the ED.

In a child, foul smelling vaginal discharge is highly suggestive of vaginal foreign body.  The most common vaginal foreign body in a child is retained toilet tissue. You can most easily remove the foreign body with saline irrigation.

Most common organism causing mastitis is staph aureus.

Consider HELLP syndrome in every patient >20 weeks pregnant with RUQ pain.   LDH will be elevated in HELLP syndrome. 

Snip20170920_10.png

Tubo Ovarian Abscess is usually initiated by GC or Chlamydia and as the infection progresses it becomes polymicrobial.  Treat with broad spectrum antibiotics.

Snip20170920_12.png

Side effects of post-exposure prophylactic medication for HIV include fatigue, nausea, and abdominal pain.

Snip20170924_1.png

When is radiation exposure the greatest risk to the fetus? 2-8 weeks during organogenesis.   At 8-15 weeks the fetus is prone to neurodevelopmental issues.   After 15 weeks there may be growth retardation.  However, all the ED imaging studies we do, including CT's, are under the 10rad threshold for causing teratogenic effects.

Muhammad        Pediatric Pearls

Treating a hair tourniquet

Hair tourniquets can be on multiple digits

Hair tourniquets can be on multiple digits

Cut down to the bone in the mid-line dorsal position of the digit.&nbsp; Alternate therapy is NAIR but Nair is not fool -proof.&nbsp;&nbsp; Cutting down to bone is more reliable to remove the hair tourniquet.

Cut down to the bone in the mid-line dorsal position of the digit.  Alternate therapy is NAIR but Nair is not fool -proof.   Cutting down to bone is more reliable to remove the hair tourniquet.

 Other causes of fussiness to consider in an infant are #1 sepsis,  non-accidental trauma, and uti.

Colic is another common diagnosis for fussiness in an infant less than 3 months of age.

Snip20170920_15.png

Paradoxial irritability, where the child does not want to be held, points to meningitis.

Purrulent eye drainage 3d-3weeks.  Get culture of eye drainage and treat with ceftriaxone and erythromycin to cover GC and chlamydia.

Snip20170920_17.png

Uric acid crystals in the diaper can look like blood in the diaper. It will be heme neg.  It has a salmon color. No need to get a UA, it is benign. Encouraging parents to increase child's fluid intake may lessen this finding.

Uric acid crystals in diaper.&nbsp;

Uric acid crystals in diaper. 

Williamson   Physical Wellness

Take the time to see your doctor and to see your dentist!   We as docs don't take the time to do this.  It is important.

Sleep deprivation has major physical and cognitive impacts.  It will negatively affect your interpersonal relationships. So prioritize your sleep!  Sleep is the #1 factor to a person's wellness.

Snip20170920_20.png

Eat food, mostly plants, not too much.  That is eat real food, not food-like substances. Don't drink your calories.  Plant-based foods should predominate in your daily food intake.  Watch your portion size. Leave the table a tad bit hungry. It takes your body 20-30 minutes to feel full and if you eat real fast you may overeat before you feel full.

Don't eat anything with more than 5 ingredients or with any ingredient you don't recognize.

You don't need tons of exercise to be healthy.  150 minutes a week of moderate exercise or 75 minutes of vigorous exercise a week is recommended.

Snip20170924_3.png

 

Exercise is the #2 most important factor to a person's wellness.

 

 

 

Conference Notes 9-13-2017

SurvivED      Windy City EM

This week's Conference was a City -Wide meeting of all the EM Programs in Chicago.  Each Program presented one great "Save"  I will give just a few learning points from the day.

ACMC's own Mitch Lorenz      

 Severe hypokalemia and hypomagnesemia from GI losses can cause prolonged QT and torsades.  Torsades can result in cardiac arrest.  Keep severe electrolyte abnormalities such as hyperkalemia, hypokalemia, and hypomagnesemia in your differential for patients with arrhythmias.

Editors note: Another setting that can result in profound hypokalemia and malignant arrhythmia is when treating DKA with IV insulin and the potassium level is not carefully managed. 

Mitch Lorenz represented ACMC EM in outstanding fashion at the Windy City EM meeting.

Mitch Lorenz represented ACMC EM in outstanding fashion at the Windy City EM meeting.

Everyone else:

In hypotensive patients, the bedside ultrasound is your friend.  You can't see pericardial tamponade on CXR and it is hard or impossible to diagnosetamponade with physical exam.  Ultrasound at the bedside will drastically shorten the time to diagnosis of pericardial tamponade.  In the setting of hypotension it can also identify tension pneumothorax, a big RV due to PE, cardiogenic shock, intra-peritoneal blood, and AAA.

If you do identify a pericardial effusion in a patient with chest or back pain consider CTA of the chest to evaluate for aortic dissection.  Proximal aortic dissections can cause pericardial tamponade.

Snip20170915_1.png

All the presented cases emphasized the importance of continually re-assessing your patients.

There is some data and anecdotal experience that ant/post placement of defib pads is more effective than shoulder/apex placement.  There was great save case discussed that hinged on placing the defib pads in the anteror/posterior position.  We may want to consider doing this in all cases.

Exerpt from Supporting Reference: Botto, BMJ Heart 1999;82:726–730

This reference discusses AFIB but the message I think would be similar for any arrhythmia.  CONCLUSIONS An antero-posterior defibrillator paddle position is superior to an antero-lateral location with regard to technical success in external cardioversion of stable atrial fibrillation, and permits lower dc shock energy requirements. Arrhythmia duration is the only clinical variable that can limit the restoration of sinus rhythm.

Patients who have experienced violence will internally feel severe anxiety, anger, and hyper-vigilance or awareness for future violence.  Healthcare providers can alleviate this somewhat by speaking in a friendly manner with the patient prior to delving into the medical issues at hand. Ask permission to examine them before touching their body.  The patient appreciates being treated as a person and having some small talk prior to doing the H&P.

 

 

Conference Notes 8-23-2017

Ryan/Traylor     Oral Boards

Case 1. 65 yo male with right flank pain.  He has a fever and is borderline tachycardic. Patient has history of lymphoma and is on Chemotherapy.   On exam, patient has a herpes zoster type rash on right flank and multiple other areas of his body. 

Diseminated Herpes Zoster (DHZ) Rash.&nbsp; It is bilateral and is not localized to a single dermatome. The condition is called disseminated herpes zoster (DHZ) when more than 2 contiguous dermatomes are affected, more than 20 vesicles are observed …

Diseminated Herpes Zoster (DHZ) Rash.  It is bilateral and is not localized to a single dermatome. The condition is called disseminated herpes zoster (DHZ) when more than 2 contiguous dermatomes are affected, more than 20 vesicles are observed outside the initial dermatome, or involvement is systemic. DHZ is rare and most frequently occurs in immunocompromised patients.

The patient has neutropenia on CBC.  IV acyclovir was started 10mg/kg Q8 hours.  Patient was also given antibiotics for neutropenia.

Airborne precautions: Airborne droplet nuclei measuring ≤5 μm can remain infective and suspended in the air for hours at a time, particularly in enclosed and poorly ventilated spaces. Airborne transmission of tuberculosis,63-65 measles,66,67 and severe acute respiratory distress syndrome (SARS)68–70 has been described in ED settings. Varicella(including disseminated zoster), highly pathogenic influenza, and smallpox may also be transmitted in this manner. Rapid identification and isolation of ED patients suspected of harboring an airborne disease hinges greatly upon heightened clinical suspicion, as in the case of tuberculosis.71Proper HCP protection against airborne droplet nuclei requires use of either an N95 or powered air purifying respirator. (Liang, Ann Emer Med 2014)

Case 2.  19yo female presents with right side abdominal pain.  She has fever and tachycardia. UCG is negative.  On history, patient has abnormal vaginal discharge.  Exam demonstrates RUQ tenderness as well as CMT and bilat adnexal tenderness on pelvic exam. 

Liver enzymes are not diagnostic.&nbsp; They may range from normal to markedly elevated.

Liver enzymes are not diagnostic.  They may range from normal to markedly elevated.

Treatment should include

  • Doxycycline + cefoxitin or cefotetan

  • Alternatives include gentamicin + clindamycin; or ampicillin/sulbactam + doxycycline

  • Continue parenteral antibiotic administration for 24 hr after clinical improvement, then switch to oral antibiotics to finish 14 day course

  • Laparoscopy can be used to lyse adhesions in the acute and chronic stages of Fitz-Hugh–Curtis syndrome

  • Add metronidazole when anaerobes are a particular concern

Case 3. 4 yo male with left hip pain and won't bear weight.  No fever. Other vitals are normal. History reveals patient had a mild viral illness a week prior.   X-rays are negative.  Labs are all normal. Diagnosis is toxic synovitis. 

You can differentiate toxic synovitis from septic hip by normal CBC, ESR, and CRP and no fever.  You should also get X-rays of the hip.   Harwood comment: Assessing the response to NSAID's is really a critical action.  If the child can walk after oral NSAID's that is a key finding to ruling out septic joint. 

2011 Singhal &nbsp; Journal of Bone and Joint Surgery Br.&nbsp;&nbsp; If a child has a normal CRP and can bear wieght, he/she has a low risk of septic joint.&nbsp; Abscence of fever and low WBC count also help differentiate but are less predictive.

2011 Singhal   Journal of Bone and Joint Surgery Br.   If a child has a normal CRP and can bear wieght, he/she has a low risk of septic joint.  Abscence of fever and low WBC count also help differentiate but are less predictive.

Toerne      ETOH Withdrawal

4 ETOH Withdrawal Syndromes include: Seizures, Uncomplicated withdrawal,  Alcoholic hallucinosis, and DT's.

Seizures: Occur early in the course of withdrawal. (within 48 hours of cessation of drinking) Commonly the first symptom of withdrawal.  These are brief, self-limited, generalized tonic clonic seizures.  

Uncomplicated Alcohol withdrawl: Hyperadrenergic vital signs and tremor.  There is no altered mental status.

Alcoholic Hallucinosis: Rare syndrome.  Patients have hallucinations with otherwise intact mental status.  Ted says he is not certain this syndrome exists.

Delirium tremens:  The patient has adrenergic storm plus delirium.  Delirium defines this syndrome.  This can occur 3-10 days after cessation of drinking.

First management goal is to identify the patients in withdrawal or at risk of withdrawal.

Ted-T's key screening questions: How many days per week do you drink? When you drink, how much do you drink?

Other clues to alcohol abuse include ruddy facial complexion, swollen hands and feet, hypokalemia, hypomagnesiemia, anemia with an MCV around 105, and thrombocytopenia.

Management: Provide a quiet environment with frequent re-assessment.  Screen these patients for other medical or traumatic problems.  Administer withdrawal-specific therapy.

Ted suggests:  Give two doses of lorazepam 2-4mg about 10-15 minutes apart.  If patient still has severe withdrawal symptoms following that, give 10mg/kg of phenobarbital.  If they still have severe withdrawal symptoms after lorazepam and phenobarbital, they are headed to the MICU.   At that point start 5mg/hour lorazepam drip.  Give another 5mg/kg of phenobarbital.    If still having trouble managing withdrawal symptoms, the third line drug would be ketamine.   Ted advocates early use of phenobarbital if the initial lorazepam is not controlling the patient's withdrawal signs.

Dawson          Pediatric Sepsis

Gabby's Law requires screening protocols and severity stratification of pediatric sepsis.  It also requires reporting of pediatric sepsis in Illinois.

Remember that irritability or lethargy can be a sign of organ dysfunction (CNS).&nbsp;&nbsp; Lactate is not reliable for identifying sepsis in pediatric patients.&nbsp; If it is high, it is concerning.&nbsp; If it is normal the child may still be se…

Remember that irritability or lethargy can be a sign of organ dysfunction (CNS).   Lactate is not reliable for identifying sepsis in pediatric patients.  If it is high, it is concerning.  If it is normal the child may still be septic.

 

Risk factors for sepsis include: cancer, transplants, immunosuppression, short gut, congenital heart disease, indwelling lines and devices,  chronic medical problems.

We screen for pediatric sepsis with vital signs and a bedside huddle.

This child's vitals point to sepsis.&nbsp; Next step is a bedside huddle with other caregivers to decide if further concern for sepsis is warranted and continuing down algorithm or if the patient has an alternative diagnosis.

This child's vitals point to sepsis.  Next step is a bedside huddle with other caregivers to decide if further concern for sepsis is warranted and continuing down algorithm or if the patient has an alternative diagnosis.

In the first 20 minutes give a first IV fluid bolus 20ml/kg, start high flow O2, and get antibiotics ordered.

Snip20170823_8.png

You need to get IV fluids into a kid rapidly using a push-pull method. Give 3 20ml/kg boluses in the first hour.  Give antibiotics in the first hour.  Start high flow O2 on all suspected sepsis patients.

Snip20170823_10.png
Snip20170823_11.png

Berklehammer        GI Bleeding

Unfortunately I missed this excellent lecture.

 

Conference Notes 7-26-2017

Carlson           Pediatric Toxicology Problems

Top 5 Pediatric Exposures in Frequency

1. Cosmetics and personal care products

2. Household cleaning products

3. Analgesics

4. Foreign bodies, toys

5. Topical preparations

Poison control centers are able to manage a large number of patients at home so they don't need to present to an ED.  There has never been a case in the last 15 years of a patient managed at home by the poison center who had a bad outcome.  They are very conservative in their management and will send the patient to the ED if any doubt.   So.....Support your local Poison Control Center!

Most common causes of pediatric deaths due to toxin exposure 2015:  (N=66) 1. Analgesics (ASA, APAP), 2. Button batteries, 3. Fumes, 4. Stimulants, 5. CardioVascular drugs.

How are products tested to determine if they are child-resistant?

Panels of 50 children (42-51 months) are tested sequentially following division into three age categories (42-44 months; 45-48 months; 49-51 months). The testing period is 10 minutes and children are instructed on how to open the package and that they may use their teeth.  If test results are inconclusive, additional testing involving one or more groups of 50 children each is required. A maximum of 200 children may be tested. (Consumer Healthcare Products Assoc)

Biggest EM Toxicology Concerns for Kids: Calcium Channel Blockers, Camphor, Clonidine, TCA's, Opioids, Lomotil, Methylsalicylate, Sulfonylureas, and Toxic alcohols.

Latest thinking on management of carfentanil (100 times more potent than fentanyl)is that it is very similar to the management of heroin.  Patients don't necessarily need huge doses of narcan.  They likely will respond to normal doses of narcan.  

Carfentanil:    "The drug was never intended to be consumed by humans. But it has been used to kill and immobilize humans — reportedly, in assassination attempts and by Russian Special Forces in 2002. They apparently used it in aerosol form as a knockout gas to end a hostage situation. Tragically, the gas ended up killing more than 100 hostages.

Boos is the section chief for the Diversion Control Division, of the DEA's Drug and Chemical Evaluation section. He says the only legitimate use of the drug is as a tranquilizer for very large animals, like elephants or hippos. So there's no medical literature to consult for its effects on humans. That knowledge is being gained the hard way, by first responders."  (NPR Reference)

For button batteries in the esophogus, ear or nose, get them out in 2-4 hours.  If it is in the stomach or GI tract it can be managed expectantly with serial abdominal xrays.

Really worry about these toxins: diltiaem/verapamil, methadone, methanol, hydroflouric acid, colchicine, paraquat, amanita mushrooms, cyanide.

Worry about these toxins: beta blockers, clonidine, TCA's, MAOI's, atropine, ethylene glycol, sulfonylureas, theophylline, carbamazepine, causits, salicylates

Dont worry about these toxins: brodifacum, Chlorox bleach(3% sodium hypochlorite), ACEI, ARB, diuretics, cholesterol medications, antibiotics, OTC camphor products, Ibuprofen, H2 blockers, Actos/Avandia.

It is always OK to not give activated charcoal.   There is a 1 hour window from the time of ingestion of toxin to give the charcoal.  Outside of the 1 hour window it has little effect. It is tolerated best when mixed with chocolate syrup and drinken with a straw from a closed cup.   Never place an NG tube to give charcoal.  Never force-feed charcoal to a child.  The risk of aspiration of charcoal is significant.

Traylor     Code STEMI

Start with IV access, consider O2 (some question of benefit), place patient on a monitor, give po ASA, give IV Heparin 4000U bolus followed by 12U/kg/hr drip.   Give Plavix vs Brilenta based on Cardiolgoy preference.

PAIL is a nice mnemonic to remember reciprocal changes.  The letter following the prior letter is where the reciprocal changes will be.  Posterior MI -Anterior depression, Anterior MI -Inferior depression, etc.  

With Inferior MI's, if the ST elevation in lead 3 is greater than lead 2 you are more likely to have an RCA occlusion.

EKG suggestive of RCA occlusion.&nbsp;&nbsp; Editor note:&nbsp; A mnemonic I just made up is 3-2-1. ST elevation in 3&gt;2 and ST depression in Lead 1.

EKG suggestive of RCA occlusion.   Editor note:  A mnemonic I just made up is 3-2-1. ST elevation in 3>2 and ST depression in Lead 1.

EKG suggestive of Circumflex Occlusion.&nbsp; Editor note: My next mnemonic is 2-3-none.&nbsp; ST elevation is 2 and 3 are equal with no ST depression in Lead 1.&nbsp;&nbsp;&nbsp; So, 3-2-1 and 2-3-none are mnemonics you can use to differentiate a R…

EKG suggestive of Circumflex Occlusion.  Editor note: My next mnemonic is 2-3-none.  ST elevation is 2 and 3 are equal with no ST depression in Lead 1.    So, 3-2-1 and 2-3-none are mnemonics you can use to differentiate a RCA and Circumflex occlusion.

Wellen's syndrome is a sign of critical LAD stenosis. These patients are at high risk for Anterior MI and should not be given stress testing.&nbsp; They need a cath.

Wellen's syndrome is a sign of critical LAD stenosis. These patients are at high risk for Anterior MI and should not be given stress testing.  They need a cath.

De Winter EKG changes are considered an Anterior STEMI equivalent

De Winter EKG changes are considered an Anterior STEMI equivalent

Lovell    Occupational Wellness

The work culture and the system you work in is the largest factors contributing to burnout.  Factors in the work environment that are detrimental to wellness include loss of autonomy, and feeling undervalued by leadership or administration. 

Ways to battle burn out: 

1. Find Compassion role-models.  Try to model your behavior on people who are genuinely caring toward others.  These people can be uplifting to you in your work.   Make a connection with each human being you care for.

2. Your patients are not your enemies.  Patients are just looking for help.  They may have low health literacy.  Be their ally.  That mind frame is much more positive and better for your own wellness.

3. Foster social resilience.   You and your co-workers are really a tribe .  Being able to rely on others in your tribe is a big factor in your personal wellness as well as in your fellow tribe members.

4. Coping with medical error.    Acknowledge your own imperfection, learn and teach about mistakes, and most importantly forgive yourself.

5. Develop a growth mindset.   Your learning and knowledge evolve over time.  Learn to value feedback and even look for feedback.  Support other people's successes.     Avoid the fixed mindset where you feel your knowledge is fixed and you see other's successes are a threat.

6. Read about Medical Humanism

7. Mitigate your Unconscious Bias    You can go to Implicit.Harvard.edu and measure your implicit bias.  Try to take the perspective of people different from you.  Read a the article "How to be a real EP; Advice to new Graduates"  Roberts.  Editor note: Great article and pops up with a quick google search.

Excerpt:

Always put the patient's well-being and the family's expectations first and foremost. Everyone thinks you know far more than you actually do, so take advantage of that lovely yet secret scam, and step up and portray the Godsend they expect and want to believe you are.

Above all, always, always, always be nice. Remember, patients and family rarely remember exactly what you said, but they always remember exactly how you made them feel. There is only one time to make that first impression, a great opportunity to brand yourself as a hero and angel of mercy or a complete jerk. Be nice to the cleaning lady, security guard, cafeteria worker, and x-ray tech. And learn their names; they know yours.

In the end be kind to everyone you meet for they are all fighting a hard battle.  Ian Maclaren

Sklar    Safely Discharging ED Patients

The ED discharge process is high risk.  However, many physicians find the discharge process time consuming and not all that important. We need to re-cnsider our approach to discharging patients from the ED. 

We overestimate the patients' understanding of discharge instructions.  Only 22% of the time do we confirm that the patient understands our instructions.   Only 16% of the time do we ask if the patient has any questions.

Patients who do not understand their diagnosis and treatment plan are more likely to be non-compliant, bounce back to the ED, have increased morbidity/mortality, lower satisfaction, and increased risk of pursuing malpractice litigation.

Poor discharge planning and instruction is the #2 cause of malpractice litigation in EM. 

Factors increasing discharge risk: incomplete or misunderstood instructions, overly-fast discharge due to production pressure, discharge without reconciliation of symptoms and test results.

Discharge is really a Hand-Off to the patient's self care.  Treat discharge time similar to how you sign out patients to another physician.  Take the time and care to discuss with the patient and their family what they need to do at home to get better.

The majority of malpractice cases involve a patient who was discharged home. The emergency physician should have a template or checklist for safely discharging patients.

mnemonic WTF DR DC:

What we found, Treatments, Follow up plan, Drugs, Restrictions, Diagnosis, Come back if.....

mnemonic as a Discharge TemplateWTF DR DC?

mnemonic as a Discharge TemplateWTF DR DC?

AMA is a high risk ED discharge situation.&nbsp; The above points are all very important.

AMA is a high risk ED discharge situation.  The above points are all very important.

Great summary of using the discharge time period to make one last re-evaluation of the patient, their vitals, and their diagnostic testing results.

Great summary of using the discharge time period to make one last re-evaluation of the patient, their vitals, and their diagnostic testing results.

 

Walesa      Infection Control Update

Bottom line: Wash your Hands. It is the #1 way to prevent the spread of infections.   Minimize your use of urinary catheters. 

Hospital acquired infections are common (75,000 patients per year), potentially deadly, and very costly.

There are more bacteria in your mouth than there are people on the earth.

A handshake transfers 124 million bacteria between the two people.

Fingernails, thumbs, and webspaces are the spots most commonly missed when washing hands.

Hand sanitizer kills bacteria better than soap and water, except norovirus and c-diff.

When placing a central line, the chlorhexadine prep needs to dry to kill the bacteria.

 

 

Conference Notes 7-19-2017

Girzadas/Marshalla     Oral Boards

Case 1.  29 you female extricated from an MVC.  Patient is 35 weeks pregnant. Patient has abdominal pain and vaginal bleeding.  She is hypotensive.  Fast exam reveals no intra-abdominal bleeding. Pelvis is stable.  Diagnosis is traumatic placental abruption.  Management: With C-spine precautions, tilt patient to left side to displace uterus off the vena cava.  Replace volume loss with crytalloid and blood products.   Initiate massive transfusion protocol.  Monitor the fetus.  Patient needs to go for emergent c-section. 

Abruption can cause DIC which may necessitate FFP, Platelet, and or Cryopreciptate transfusion.  Consensus among the faculty was that TXA would also be indicated in this case.

U/S is insensitive for diagnosing placental abruption.  Fetal monitoring is more sensitive.  Have a low threshold in pregnant women with a viable pregnancy who suffer a fall, mvc, or other trauma to send the patient to L&D for monitoring.   The placenta is inelastic.  With trauma or deceleration the uterus will deform and the placenta won't causing a shear stress that results in abruption.

The initial symptoms of abruption can be mild or overlooked.  The standard fetal monitoring period is 4 hours.  If the monitor shows infrequent contractions and normal fetal heart rate, the patient can be discharged after 4 hours.  If the patient has more frequent contractions or a worrisome fetal heart tracing the patient will be kept for a long period of time.

 

Case 2.  31 yo man with palpitations.  Initial EKG shows fast, irregular rhythm with varying QRS intervals.

Elise comment: Know this EKG.&nbsp; Irregular, wide and narrow, fast.&nbsp; The answer for this EKGis always procainamide if stable, synchronized cardioversion if unstable.

Elise comment: Know this EKG.  Irregular, wide and narrow, fast.  The answer for this EKGis always procainamide if stable, synchronized cardioversion if unstable.

Patient then became unstable with hypotension and altered mental status.  EKG now shows:

Snip20170720_2.png

WPW with AFIB has degenerated to a wide complex, very fast tachycardia.  With the patient being unstable, immediate synchronized cardioversion is indicated.

EKG after cardioversion shows WPW syndrome with clear delta waves.

Patient was admitted to cardiology service for catheter ablation.

Case 3. 6 yo child with injury and gross deformity of left elbow after a fall from bike.  Xrays show displaced supracondylar fracture (Gartland 3)

Keys to management include careful neuro-vascular exam of injured extremity.  The most commonly injured nerve with supracondylar fractures is the median nerve.  A pure motor branch of the median nerve that is commonly injured is the anterior interosseus nerve.  This nerve is tested by seeing if the patient can perform the OK sign.

Supracondylar fractures have high risk for vascular (brachial artery) injury, nerve (median and radial) injury, compartment syndrome, and chronic boney deformity (cubitus varus).  Untreated, compartment syndrome can result in volkmann's ischemic contracture.

Non-displaced supracondylar fractures with intact NV status can be splinted and dc'd home with close Orthopedic follow up after consultation with Orthopedic Specialist.  Class 2 fractures with an intact posterior cortex will be treated on a case by case basis per Orthopedic surgery.  Some may be able to be discharged, some may need to have ORIF and admission.  All Class 3 fractures are going to the OR. If the patient has a good pulse and warm fingers there is no need for emergent manipulation of the fracture in the ED.  If the pulse is diminished and the fingers are warm, have cap refill, and/or have a good pulse ox wave form then again there is no need for emergent ED manipulation of the fracture.  This patient can be managed in the OR.  If the pulse is diminished and the fingers are cool or have poor cap refill or have poor pulse ox waveform and there will be a delay to going to the OR then after discussion with the Orthopedic Specialist, the emergency physician may need to attempt reduction of the fracture prior to the patient going to the OR.

 

Williamson      Study Guide      GI Emergencies

American College of Gastroenterology Guideline for Diarrhea Management 2016

American College of Gastroenterology Guideline for Diarrhea Management 2016

Treat traveler's diarrhea with cipro or azithromycin.  Traveler's Diarrhea has high enough risk of bacterial pathogen to warrant antibiotics.  Below are recommendations by the American College of Gastroenterology

Viruses are the most common cause of diarrhea. Campylobacter is the most common bacterial cause of diarrhea presenting to the ED.  

Most common cause of food poisoning is staph gastroenteritis.

Glucagon showed no benefit over placebo for treatment of esophageal food impaction.

Crohn's and Ulcerative Colitis

Crohn's and Ulcerative Colitis

Crohn's and Ulcerative Colitis

Crohn's and Ulcerative Colitis

Ogilvie's syndrome can be treated with neostigmine.  Alternative treatment is colonoscopic decompression.

Ten of the 11 patients who received neostigmine had prompt colonic decompression, as compared with none of the 10 patients who received placebo (P<0.001). The median time to response was 4 minutes (range, 3 to 30). Seven patients in the placebo group and the one patient in the neostigmine group without an initial response received open-label neostigmine; all had colonic decompression. Two patients who had an initial response to neostigmine required colonoscopic decompression for recurrence of colonic distention; one eventually underwent subtotal colectomy. Side effects of neostigmine included abdominal pain, excess salivation, and vomiting. Symptomatic bradycardia developed in two patients and was treated with atropine.  (NEJM 1999 341:137)

Anthony Gallaway Equality Illinois    LGBTQ Ally Development Training

Gender identity does not define Sexual Orientation and Sexual Orientation does not define gender identity.  

Sex assigned at birth does not define gender identity or gender expression or sexual orientation.  

As a physician, sometimes it is medically necessary to delve into a patient's gender status.  If there is some uncertainty on the physician's part about a patient's gender (biologic, identity, expression),  one suggested strategy is to introduce yourself and say these are my pronouns (she, her, her's, or he, him, his). You can then ask the patient, "what are your pronouns?"    If indicated to appropriately treat the patient, you can ask, "What gender were you assigned at birth?"    If medically necessary, you can ask further, "Do you have sexual relations with males or females or both?" If a genital exam needs to be performed, be sure to fully inform the patient what will be done as part of the exam.   Throughout all this history and physical process, it is most important to be supportive and caring. Demonstrating that you are an ally will help the patient the most.  Only delve into these questions if it is important to care for the patient medically.  If the answers to these questions are not directly relevant to the patient's care, don't ask them. 

Intention is everything.  If you make some mistakes while being well-intentioned and caring it is OK.

To re-emphasize, as a clinician you don't always need to know the patient's sexual assignment at birth or their gender status.  So you definitely don't always need to ask those questions.

LGBTQ patients have fear or discomfort with medical care because they don't know if the medical providers are going to be supportive or judgmental/disapproving of them.

A person can take Truvada with use of a condom to prevent HIV infection. This is called PREP (pre-exposure prophylaxis).

It means so much to LGBTQ patients to express a supportive and welcoming attitude.  Ask, "How can we help you today?"  Use your smile and your warmth when you ask this.

Ohl     Sedation and Paralytics

Sean discussed the sedation medications and neuromuscular blockers used for Rapid Sequence Intubation.

Sean then discussed the key points to rapid sequence intubation. 

Wing       EICU Process for Patients Awaiting a MSDU Bed

Elisa discussed the new process for initiating EICU coverage for patients boarding in the ED while waiting for step-down beds.

 

Conference Note 7-12-2017

Yappo         Review of 2016 AAP Practice Guideline for BRUE

BRUE= Brief <1 min, Resolved, Unexplained, Event in an infant less than 1 year old. The event needs to have 1 or more of the following: apnea, pallor, change in breathing, change in tone, or decreased responsiveness.  By the time the child is in the ED the exam must be normal to be a BRUE.

You can do a quick evaluation of an infant's growth by looking for a previous ED visit. If the patient was in the ED for a prior visit you can get their weight at that time.   You then plot out that weight and the weight from the current visit to see if the child is appropriately gaining weight.   You can also just check the child's current weight against expected weights on the growth curve.  If a child is significantly below the growth curve in the setting of BRUE, you may need to consider abuse, neglect, or metabolic problems. An under-weight infant would take the patient out of the low-risk BRUE category.   Additionally, head circumference larger than normal on the growth curve may be a marker of non-accidental head trauma.

For BRUE, the apnea described by caregivers should not just be peripheral cyanosis.  For BRUE criteria, cyanosis should be central cyanosis. 

Central Cyanosis

Central Cyanosis

Low Risk BRUE: Born after 32 weeks, >60days of age, no CPR required by a medically trained provider, event lasted less than 1 minute.

High Risk BRUE: Prior BRUE, history of congenital heart disease or inborn error of metabolism, family history of BRUE or sudden death, bruising or other signs of trauma, abnormal growth, abnormal vital signs.

Management of BRUE.&nbsp; If you are going to do some tests on low-risk BRUE patients, consider an EKG, it has good negative predictive value if normal.&nbsp; Pertussis testing is another consideration based on immunization status and seasonal preva…

Management of BRUE.  If you are going to do some tests on low-risk BRUE patients, consider an EKG, it has good negative predictive value if normal.  Pertussis testing is another consideration based on immunization status and seasonal prevalence.  Infants with pertussis may have gasping or apnea prior to cough or other respiratory symptoms.   Monitor the child for 4 hours in the ED with pulse ox and document repeated exams.  No need to search for occult infection unless the child is less than 2 months or appears ill.  Same with inborn errors. No need to look for inborn errors unless child is <2months, ill-appearing, or has family history of BRUE or sudden death.

Editors note: BRUE reminds me of the practice guideline for bronchiolitis.  The goal is to minimize treatment and interventions unless history and physical exam indicate a need to evaluate or manage further. 

Elise comment: Identifying the low-risk BRUE is challenging. You can't do this off the top of your head in the ED.  You need to pull up the guideline and go step by step through it in real time to be sure you are not mischaracterizing a high-risk child into the low-risk group. 

Marshalla   EKG Basics

A quick way to determine the rate is to divide 300 by the number of big boxes in the RR interval.

A quick way to determine the rate is to divide 300 by the number of big boxes in the RR interval.

Normal Intervals

Normal Intervals

Quick way to determine axis based on QRS orientation in leads 1 and AVL.

Quick way to determine axis based on QRS orientation in leads 1 and AVL.

Lambert   U/S Basics   Image Acquistion & Instrumentation

The brighter an object appears on the screen the more reflective it is to sound waves.

Sound waves travel slowly through air compared to the speed of sound through tissue.  Air causes significant image degradation due to the slow speed of sound in air.   Examples:  bowel gas will make it difficult to image the aorta.  The lung will hinder imaging of the heart.

 

Anatomic Planes and Axes

Anatomic Planes and Axes

Imaging planes

Imaging planes

Low frequency sound waves travel deeper into the tissue than high frequency sound waves.  You can adjust the frequency of the probe to optimize your image in relation to the patient's BMI.  There is an easy button our ultrasound machines with pictures of a thin man and a thick man.  Click on the picture that most closely represents your patient. That will optimize the frequency for the patient's body habitus.

Lambert     Bedside Echo

The main 3 views you will use at the bedside are the subcostal, parasternal, and apical views.

U/S Images from the 4 basic views

U/S Images from the 4 basic views

Subcostal 4 chamber view is the best view for identifying pericardial effusion.

Mike said that if you see fluid between the liver and the heart on the subcostal view, it is always abnormal.

Mike said that if you see fluid between the liver and the heart on the subcostal view, it is always abnormal.

When evaluating for PE with echo, you basically are looking for a big RV.

Big RV on Left side image.&nbsp; Normal RV on Right side image

Big RV on Left side image.  Normal RV on Right side image

Lambert        Central Venous Access

Most operators use a transverse view to image the vessel. Standing at the head of the bed with the screen facing you, the operator, make sure your probe indicator is matched up with the indicator on the screen. They should both be directed to the left.

You want to approach the vessel at a 45 degree angle. So puncture the skin at the same distance from the center of the probe as the depth of the vessel to the probe. It is important to identify the tip of the needle to know where you are at. 

A has to equal B to make the 45 degree angle with the needle approach.

A has to equal B to make the 45 degree angle with the needle approach.

You can move the probe proximally and distally from the puncture site to identify the needle tip. Once you have identified the needle tip tenting the IJ vessel, make a small jab with the needle to puncture through the vessel wall and obtain blood return.   Once you pass the guidewire, use the probe in a longitduinal orientation to verify that the guidewire is in the IJ.   If you verify the quidewire is in the IJ, then you can confidently use the central line right away before obtaining a CXR.

Ultrasound Lab    Lambert and Team Ultrasound

 

Conference Notes 7-5-2017

Lovell/Ohl      Oral Boards

Case 1. 35yo male fell off a boat and suffered a severe laceration to his mid thigh from a propeller injury. 

  • Patient has severe bleeding and a tourniquet is applied to the left thigh by EMS. 
  • Massive transfusion protocol initiated in ED. 
  • Patient also has signs of worsening hypoxia due to drowning.  Patient was then intubated. Patient was evaluated for other injuries. 
  • TXA was given for severe hemorrhage.
  • Patient went to the OR for femoral artery injury. 

Do not explore the wound locally. This patient has hard signs for arterial injury and is going to the OR.

In accordance with the ILCOR guidelines, patients should be referred to as drowning victims if they have suffered a suspected respiratory injury following submersion in a liquid medium, regardless of their clinical status, which may vary from essentially asymptomatic to severely ill at time of presentation. Additional descriptors such as whether there was a precipitating event that led to drowning or whether the drowning was witnessed may be used as necessary. The primary outcome of a drowning episode is either death or survival. Adopting this clinical nomenclature will allow future studies to better characterize, study, and risk stratify drowning victims.   EM Reports Vol 16 N. 32015.

Hard and Soft signs of arterial vascular bleeding

Hard and Soft signs of arterial vascular bleeding

Case 2.  20 yo female with a headache for a few hours.  Normal Vital signs. Patient has nausea and left anterior neck and face pain as well.   Patient was on a roller coaster ride prior to the onset of this pain.   On exam patient has a horner's syndrome on the left side ipsilateral to where she is having pain.

Horner's syndrome on the left. &nbsp; Horner's caused by internal carotid artery dissection will have ptosis and miosis but not anhydrosis.&nbsp; Anhydrosis is caused by sympathetic ganglia around the external carotid, not internal carotid.&nbsp;

Horner's syndrome on the left.   Horner's caused by internal carotid artery dissection will have ptosis and miosis but not anhydrosis.  Anhydrosis is caused by sympathetic ganglia around the external carotid, not internal carotid. 

CTA of the neck shows that the diagnosis was internal carotid artery dissection. 

  • Treatment for extra-cranial dissections is usually anticoagulation. 

Case 3.  28 yo male with joint pain in bilateral wrists/hands and ankles. Patient has a temperature of 38 and otherwise normal vital signs. Patient has a few pustules on his hands and ankles.

Pustule from disseminated ghonorrhea

Pustule from disseminated ghonorrhea

  • Treatment is parenteral ceftriaxone for a minimum of 7 days.   Treat chalmydia presumptively as well.   

Dissemenated ghonorrhea can present with one of two syndromes: 1. dermatitis-tenosynovitis or 2. oligo arthritis.  Get an RPR and HIV test on every patient.  It is tough to make this diagnosis so swab urethra/cervix. If you can get fluid from a pustule or a joint also culture and gram stain that fluid.

Schmitz    M&M  

No case specifics, just a couple of take home points.

Get a CT scan of the head in patients who are intoxicated and have suffered head trauma.   If the patient refuses the study you have to carefully balance the patient's decisional capacity with the risk of intracranial injury.  Err on the side of imaging and sedate if necessary to get the study done.  Observation in the ED in place of imaging can be problematic for many reasons so getting a scan is the more fail-safe approach.

Be aware of anchoring bias.  Patients triaged to the hallway can still have serious injuries.

Be cautious of your cognitive biases.  Cognitive bias can mislead us. 

Be careful not to attribute a patient's behavior to some personality or character flaw rather than to their illness, pain, or injury. This bias is called the "Fundamental Attribution Bias"

Be careful not to attribute a patient's behavior to some personality or character flaw rather than to their illness, pain, or injury. This bias is called the "Fundamental Attribution Bias"

Chiefs      Codes (44, Sepsis, STEMI, Stroke)  in the ED

Approach all these rapid response codes in the ED the same way every time.  Assess the patient's airway.  Evaluate their breathing and circulation.  Do a rapid NIH stroke scale and expose them completely to look for other problems.    In short, ABCDE's, IV, O2, Monitor, Dexi, EKG on all these patients.

35% of patients with sepsis progress to septic shock.  So if you identify sepsis, call a CODE SEPSIS.  It will get you nursing and pharmacy help for the care of that patient.

Our Goal is administering IV antibiotics within 1 hour of identifying sepsis.&nbsp; If the patient has hypotension (systolic BP&lt;90 or MAP&lt;65) or a lactate of 4 or above they need 30ml/kg of IV crystalloid fluids.&nbsp; If you have concerns tha…

Our Goal is administering IV antibiotics within 1 hour of identifying sepsis.  If the patient has hypotension (systolic BP<90 or MAP<65) or a lactate of 4 or above they need 30ml/kg of IV crystalloid fluids.  If you have concerns that the patient cannot handle 30ml/kg of crystalloid you can alternately give 126ml/hr or higher of IV fluids and write a note in the chart discussing briefly why you felt 30ml/kg was unsafe for the patient. 

For CODE Strokes you need to do an NIH Stroke Scale.   Girzadas comment:  The NIH Stroke Scale is the new defacto EM Neuro Exam.  You need to do this on your patients with neurologic symptoms.

NIH Stroke Scale

NIH Stroke Scale

Absolute contraindications to TPA for Stroke.&nbsp; Take a close look on this chart at the factors regarding bleeding diathesis.&nbsp;

Absolute contraindications to TPA for Stroke.  Take a close look on this chart at the factors regarding bleeding diathesis. 

Traylor      Ventilator Management

Indications for intubation: 1. protect airway,  2. inadequate oxygenation and/or inadequate ventilation despite non-invasive O2 support, 3. Respiratory fatigue or anticipated respiratory failure.

Ventilator Lung Protective Strategy &nbsp;

Ventilator Lung Protective Strategy  

Obstructive Ventilation Strategy &nbsp; EMCRIT reference

Obstructive Ventilation Strategy   EMCRIT reference

Denk    Management of Shock

4 types of shock.

4 types of shock.

Treat hypovolemic shock with volume (crystalloid or blood products)

Treat with distributive shock with volume and pressors

Treat obstructive shock with volume and concurrently a needle, chest tube, impella device, thoracotomy, or TPA.

Treat cardiogenic shock with dobutamine/norepinepherine and concurrently activate cath lab for PCI and or mechanical support device.

 

 

Conference Notes 6-14-2017

Felder/Chastain/A. Patel/Tekwani/DenOuden    Panel Discussion    Efficiency in the ED

Prior to Seeing the Patient

Before you see the patient, do a brief chart biopsy to identify their basic diagnoses and identify who their PMD is.  Also, find the EKG before you see the patient.

Conversations with admitting physicians and consultants

When talking with consultants and admitting physicians, keep the conversations as brief as possible.  Using perfect serve to text the initial info to the physician can be helpful. When you speak with an admitting doctor or consultant give them a brief synopsis of the case and give them what you think needs to be done during this admission. It may be helpful to keep a note of the patient's name, MR#, and one line on their case to be prepared when the consultant calls.

You only need to speak with a consultant if you have a clinical question that needs an answer acutely in the ED or if the patient has an acute management need from this consultant.

Many panelists commented on the excellent utility of Perfect Serve when contacting consultants and admitting physicians.

Interactions with Nurses and Techs

Make sure the nurse knows the plan and the to-do tasks for each patient so they can focus and streamline care as well.

Utilize the ED Techs to help you get tasks done.

Management of the Patient

Do your best to make a diagnostic plan and place orders all at one time and avoid adding on tests later.

Always be continually running your list to check what you can do to move patients along in the ED process. Always be alert to delays with labs and imaging.  If you are always alert to where each patient is in their ED work up you can be ready to sign out at any time.

Put your discharge instructions right after you talk with the patient initially.  That is the time when you will best recall your conversation with the patient and what your plan is.

Work to find the rate-limiting step for each patient and solve that rate-limiting step.

Interactions with Patients

Always ask the patient what they are worried about or what diagnosis they think they have.  Most patients google their symptoms and have preconceived concerns about what they have going on.  If you can specifically address their concern, you have a better chance to achieve higher satisfaction.

One strategy to end the conversation with the patient is "I am going to step out now and order your tests and get your work up started.  I will be back to check on you later."

Another closing strategy is to summarize with the patient what you perceive their diagnostic problem is how you are going to work it up, what the treatment plan in the ED will be,  and what the likely dispo will be. 

Schmitz     Safety Lecture     Sepsis

For the entire Advocate System, mortality for sepsis has decreased by close to 25% between 2015 and 2017!

Advocate is starting to gather data on patients that were seen as outpatients within 72 hours of an ED visit for sepsis.  The system is trying to find ways to earlier identify infections that can develop into sepsis.

Schroff      5 Slide Follow Up

Subclavian Steal &nbsp; Because of a stenotic subclavian artery, exercise with the affected (left) arm steals flow from the vertebral artery which can result in syncope or neurologic symptoms.

Subclavian Steal   Because of a stenotic subclavian artery, exercise with the affected (left) arm steals flow from the vertebral artery which can result in syncope or neurologic symptoms.

Hart/Regan      Ortho Cases

EM Boards Classics: Montaggia and Galeazzi fracture dislocations. You gotta know these.&nbsp; Montaggia is fracture of the proximal ulna with dislocation of the radial head.&nbsp;&nbsp;&nbsp; Galeazzi is fracture of the distal radius with dislocatio…

EM Boards Classics: Montaggia and Galeazzi fracture dislocations. You gotta know these.  Montaggia is fracture of the proximal ulna with dislocation of the radial head.    Galeazzi is fracture of the distal radius with dislocation of the radio-ulnar joint.

This is a distal biceps tendon rupture.&nbsp; The muscle is bunched up superiorly.&nbsp; The proximal bicep tendon can also rupture and result in the muscle bunching up inferiorly

This is a distal biceps tendon rupture.  The muscle is bunched up superiorly.  The proximal bicep tendon can also rupture and result in the muscle bunching up inferiorly

Best Conference Attendance for the Year

#1 Jeff Florek #2 Anita Schroff#3 Jenny Denk

Congrats to Jeff, Anita, and Jenny!!! Outstanding effort this year!!!!!

Garrett-Hauser     Ethics

Steps to taking custody of a child: #1 Call the Hospital Administrator to notify them.  #2 Call DCFS #3 Discuss with the parent. You may need security on hand when you have this conversation.  These are very high risk situations. Utilize all your hospital resources: risk management, chaplain, security, nursing, and if needed, the police.

How do you change the code status for wards of the state?   The most straight forward way is to call the phone number of the patient's guardian listed on their paperwork and and discuss the management of the patient with the guardian.

For developmentally delayed patients who are not kids or elders and have a need for placement in a NH or safe environment, utilize the care managers.  They have the skill set to get the patient placed in a safe environment.  PLOWS is the state agency that helps to place adult patients who do not fit into the pediatric or elderly categories.

Regan and Alexander     The Last Lecture

Very clever, heart-warming, and funny reminiscing of the Class of 2017.  

ALL the BEST to the Class of 2017!  

 

 

Conference Notes June 7

Kettaneh/Destefani    Oral Boards

1. 65 yo male with severe vomiting. Initial vitals normal except for tachycardia.

Boerhaave's syndrome can have findings of minimal mediastinal air such as this CXR or large pleural effusion and even pneumothorax as discussed in this case.&nbsp; Esophogus usually ruptures at the left, posterior aspect. Thus you should look to the…

Boerhaave's syndrome can have findings of minimal mediastinal air such as this CXR or large pleural effusion and even pneumothorax as discussed in this case.  Esophogus usually ruptures at the left, posterior aspect. Thus you should look to the left side of the thorax for air and fluid on CXR.

Treatment was thoracic surgical consultation, IV fluid resuscitation, broad spectrum IV antibiotics, and chest tube to drain large pleural effusion.

2. 54 yo female with URI symptoms and headache for 5 days. Vitals are normal except for mild increased respiratory rate and fever.  Diagnosis was meningitis. Give steroids before or at the time of antibiotics. IV steroids have shown the most treatment effect for meningitis due to Strep pneumo.  Strep pneumonia is the most common bacterial cause of meningitis in adults.  Give vancomycin and ceftriaxone.  Consider ampicillin for listeria in patients over age 50.  Consider acyclovir for herpes encephalitis.    Nick and Andrea made the same comment: Strep meningitis may present with mild symptoms initially. So you have to keep your guard up. 

Graph showing the incidence of different etiologic agents of meningitis base on age. Strep pneumo is predominant after for patients 19yo and up. Listeria is increased at less than 1 month and over age 60.&nbsp;

Graph showing the incidence of different etiologic agents of meningitis base on age. Strep pneumo is predominant after for patients 19yo and up. Listeria is increased at less than 1 month and over age 60. 

 

3. 5 yo male with URI symptoms for 5 days. Vitals are normal.  Patient has had fever during the last 5 days.

Patient had clinical findings consistent with Kawasaki's disease

Patient had clinical findings consistent with Kawasaki's disease

Diagnostic criteria for Kawasakis Disease.&nbsp; It is also called muco-cutaneous lymph node syndrome.&nbsp; This term is a brain hack to remember the criteria for diagnosis.

Diagnostic criteria for Kawasakis Disease.  It is also called muco-cutaneous lymph node syndrome.  This term is a brain hack to remember the criteria for diagnosis.

ESR and CRP are usually elevated in the setting of Kawasaki's.  Treatment is po ASA and IVIG.

Carlson      Toxicology Axioms for the EM Graduate

Toxicologic Mimics

Iron toxicity, ASA, and toxic alcohol poisoning can mimic DKA

CO, arsenic, and digoxin toxicity can mimic food poisoning

CO poisoning can mimic SAH hemorrhage

Salicylate overdose can mimic meningitis

Cyanide, CCB's and ASA overdoses can mimic sepsis

Toxidromes

Toxidromes

CCB's, Beta blockers, Organophosphates, botanical cardiac glycosides, flourides, clonidine, sodium channel blocking drugs, INH and gyromitra will all need very large doses of antidote.  Carfentanyl also requires very high doses of naloxone. Most protocols for treating carfentanyl overdoses recommend early intubation to avoid using up stockpiles of narcan.  

 

Gastric lavage is rarely necessary to manage an overdose.  Charcoal administration is also very infrequently required to manage an overdose. If you have any concerns about vomiting or aspiration, you can feel OK about not giving charcoal. 

Get serial levels of salicylate, vlaproate, lithium, tegretol, and theophylline if you have concerns that any of these substances could have be taken in overdose.

Even if you aren't sure of the overdosed medication, many time you can still treat effectively. This is because most of toxicology is supportive care: protect the airway, treat hypotension, cool the hyperthermic patient, correct acid/base abnormalities, prevent renal injury, dialyze as needed, and protect against self-harm.

Williamson      "So you got a Subpoena"

This lecture is not about medical malpractice but rather about subpoenas physicians receiveto testify as a medical professional.

If you receive a subpeona for medical records, refer the request to risk management.

3 types of cases you may be asked to testify: civil, criminal, DCFS.   For civil cases provide testimony on the substance of treatment, the patient's medical condition, and their prognosis.   For criminal cases, you may be asked to provide medical testimony regarding a crime.  DCFS may subpoena you to testify regarding your clinical findings about a child who may have suffered abuse or neglect.

Always contact risk management when you get a subpoena.  Check with your hospital whether they provide legal counsel for treating physician testimony that is not medical malpractice related. Most hospitals do not offer legal counsel for non-malpractice issues.  However, if they do, take advantage of it. It is always helpful to have a lawyer advising you.

The scheduled time on the subpoena is not accurate.  You need to contact the law office to find out the real time.  There is usually some flexibility regarding the date and time of your testimony that you can negotiate with the lawyer.  For depositions, you can ask for a location that is convenient for you.

For depositions for civil suits you should ask to be paid.   Most faculty say they ask for somewhere between $300-500 per hour.   You will be paid at the time of deposition.  When calculating your time, include your travel time.

AMG suggested fee schedule for depositions, testimony, and record review.

AMG suggested fee schedule for depositions, testimony, and record review.

There was a discussion about whether it is reasonable to review the chart prior to going to a deposition or trial.  Most faculty said they would ask for the chart through risk management and not go into the patient's medical record on their own. 

Wear a suit to the deposition or trial.  You are being judge on your appearance and statements. Many depositions are recorded on video.

Always review the transcript of your testimony.  Don't waive your right to review the transcript of your testimony.

When giving a deposition always tell the truth.  Your answer has to be right, correct, and accurate. Do not answer hypothetical questions.

Elise comment: You have to be like a rock during a deposition, emotionless.   The lawyers are trying to get you to react.  Don't take the bait.  Always, answer calmly sticking to the direct facts that are documented on the medical record.  If they are asking questions of the case beyond the medical record, you can say repeatedly "I have no independent recollection"

Dennis comment: Cautiously give as focused and as limited an answer that you can give to each question.

If you get served a subpoena for medical malpractice, contact risk management immediately and don't discuss with anyone else.

Logan    5 Slide Follow Up   20+ week Peripartum Emergencies

Peripartum Cardiomyopathy   Diagnose with echo.  Treat with nitorglycerine, diuretics, inotropes, and heparin.

Thromboembolic disease   5 times greater risk during pregnancy and 60 times the risk for 3 months after pregnancy .  Treat with heparin which does not cross the placenta.

Pre-Ecclampsia and HELLP   Diagnose with a straight-cath Protein/Creatinine ratio.   Treat with magnesium, BP control, and delivery.

Placental Abruption   Diagnose with fetal monitoring.  U/S is not sensitive for placental abruption.

Hawkins    5 Slide Follow Up    Calcium Channel Blocker Overdose

Management(Life in the Fast Lane Reference)

  • early intubation and ventilation when life-threatening toxicity is anticipated
  • Early invasive blood pressure monitoring if evolving hypotension and shock; initiate therapies below

Specific treatment (support cardiovascular system)

  • Fluid resuscitation (up to 20 mL/kg crystalloid)
  • Calcium
    • can be a useful temporising measure to increase HR and BP
    •  options
      • 10% calcium gluconate 60mL IV (0.6-1.0 mL/kg in children)
      • 10% calcium chloride 20mL IV (0.2 mL/kg in children) [must be given via CENTRAL VENOUS ACCESS – it burns!]
    • repeat boluses can be given up to 3 times
    • consider calcium infusion to keep serum calcium >2.0 mEq/L
  • Atropine: 0.6mg every 2 min up to 1.8 mg (often ineffective)
  • High dose insulin – euglycaemic therapy (HIET)
    • see below
  • Vasoactive infusions
    • titrate catecholamines to effect (inotropy and chronotropy); options include dopamine, adrenaline and/ or noradrenaline
    • if vasoplegic, consider noradrenaline and vasopressin. Consider methylene blue if refractory (to decrease cGMP formation, scavenge nitric oxide, and inhibit nitric oxide synthesis leading to vasoconstriction).
  • Sodium bicarbonate
    • consider in severe metabolic acidosis
    • 50-100 mEq sodium bicarbonate (0.5-1.0 mEq/kg in children)
  • Cardiac pacing
    • electrical capture may be difficult to achieve and may not improve overall perfusion
    • use ventricular pacing to bypass AV blockade, typical with rates not in excess of 60/min
  • Intralipid
    • consider in refractory cases, as calcium channel blockers lipid soluble agents
  • Circulatory support devices
    • consider in refractory cases
    • VA ECMO or cardiac bypass is preferred to intra-aortic balloon counterpulsation (useful if poor inotropy, will not correct refractory vasoplegia)

High-dose insulin euglycaemic therapy (HIET)

  • The place of HIET in the step-wise approach to managing cardiovascular toxicity has evolved
  • Formerly considered a last ditich measure, early is use is increasingly advocated. This is important as the beneficial effects of HIET are not immediate

 

Elise and Harwood both made the point of avoiding charcoal in any patient with altered mental status or risk of seizure, vomiting, or needing intubation.

Elise and Harwood both made the point of avoiding charcoal in any patient with altered mental status or risk of seizure, vomiting, or needing intubation.

Recommended high-dose insulin euglycaemic therapy protocol based on the clinical experience of the Western Australian Toxicology Service, published case reports, reviews and animal studies (from Nickson and Little, 2009)

Schmitz      Administrative Update

Regan/Hart   Visual Diagnosis

Cannon ball metastases are associated with renal cell carcinoma and choriocarcinoma.&nbsp; Less commonly prostate cancer, synovial sarcoma, and endometrial cancer.

Cannon ball metastases are associated with renal cell carcinoma and choriocarcinoma.  Less commonly prostate cancer, synovial sarcoma, and endometrial cancer.

Diffuse ST depression with ST elevation in AVR is consistent with Left Main coronary Artery occlusion.  

Diffuse ST depression with ST elevation in AVR is consistent with Left Main coronary Artery occlusion.

 

 

Glioblastoma classically has butterfly appearance on CT.&nbsp; See image below as well. &nbsp;

Glioblastoma classically has butterfly appearance on CT.  See image below as well.  

Glioblastoma

Glioblastoma

Conference Notes 5-31-2017

McKean/Schmitz

Case 1. Digoxin poisoning from botanical cardiac glycoside. Child was bradycardic and EKG showed bradycardia and heart block.  

2nd Degree heart block.&nbsp; 2:1 block. Can't determine whether it is Mobitz I or II based on this ECG.

2nd Degree heart block.  2:1 block. Can't determine whether it is Mobitz I or II based on this ECG.

Andrea comment: For boards, he most common plant that will cause botanical glycoside toxicity is Oleander.

Case 2. Black widow envenomation.  75% of bites are on extremities and are asymptomatic.  For patients with significant pain and vital sign abnormalities, antivenom can be a critical treatment modality.   Andrea comment: No hospitals have black widow antivenom on site.  You will need to call poison control. Poison control will get antivenom from the zoo.  Calcium is no benefit for black widow spider envenomation. Benzo's however, can symptomatically improve the muscle spasm.

Case 3. Achilles tendon rupture can be diagnosed with Thompson's Test. (96% sensitive, 93% specific).

Picture A shows normal plantar flexion with squeezing the calf=negative Thompson's Test.&nbsp; Picture B shows no plantar flexion with squeezing the calf=positive Thompson's Test.&nbsp;&nbsp; Edtor's note: I can never remember which is positive and …

Picture A shows normal plantar flexion with squeezing the calf=negative Thompson's Test.  Picture B shows no plantar flexion with squeezing the calf=positive Thompson's Test.   Edtor's note: I can never remember which is positive and which is negative.  So in the chart I write Thompson's test showed intact Achilles tendon function. Or, Thompson's test indicated Achilles tendon rupture.  For the Thompson's test to be valid you need to have the knee flexed.  So either lay the patient on their stomach and flex their knee or have them kneel with one knee on a chair or cart.

Lee   M&M

No case details, just some take home lessons.

Hypotension in association with aortic dissection indicates either the patient has an ascending aortic dissection with pericardial tamponade or the patient has a surgical indication for a type B dissection. Either way consult Cardiothoracic surgery.

Cardiothoracic surgery is the service to consult for suspected aortic dissections.

Do a bedside echo on patients on whom you have concern for aortic dissection and all critically ill patients.  Elise comment: Do serial bedside echo's to determine if pericardial fluid is accumulating.  This can help identify a dissection of the ascending aorta.   Elise Hart comment: TEE has high sensitivity for identification of ascending aortic dissection.

Dr. Lee then lead a fascinating discussion about the clinical situation of very high levels of ethanol intoxication and concern for toxic alcohol ingestion.  Very high levels of ethanol can cause death by respiratory depression, and at levels around 600, cardiovascular instability.   There are two online osmolar gap calculators when you do a google search, 1. University of Iowa and 2. MD Calc.   The University of Iowa calculator uses a different calculation than MD Calc.  It uses a conversion factor of 1.2 to account for some invitro studies that showed very high levels of alcohol affect the osmolar gap.  Andrea said that this new calculation used by the University of Iowa Calculator has not been fully validated.  She feels the MD Calc osmolar gap calculator is safer for patients.

Traditional Osmolar gap calculation used by MD Calc calculator.

Traditional Osmolar gap calculation used by MD Calc calculator.

To account for ETOH, this formula uses 1.2 x ETOH/4.6University of Iowa Calculator.&nbsp;&nbsp; Andrea felt this calculator is not ready for prime time.&nbsp;&nbsp; Harwood is going touse this calculator. He feels it cuts down on the number of patie…

To account for ETOH, this formula uses 1.2 x ETOH/4.6

University of Iowa Calculator.   Andrea felt this calculator is not ready for prime time.   Harwood is going touse this calculator. He feels it cuts down on the number of patients who need to have fomepizole and dialysis.

Serum alcohol level only measures ethanol.  Toxic alcohol ingestion has no impact on the blood alcohol level.

Chinwala     5 Slide Follow Up

Diagnostic algorithm forAnaphylaxis

Diagnostic algorithm forAnaphylaxis

To quote Dr. Chinwala, "EPI is King" when treating anaphylaxis.&nbsp;

To quote Dr. Chinwala, "EPI is King" when treating anaphylaxis. 

Hart/Regan     Ortho Updates

Segond Fracture is highly associated with ACL tear and meniscal injuries.

 

This is one of the most common ankle fractures in snowboarders.&nbsp; It was a rare high-energy type injury prior to the sport of snowboarding.

This is one of the most common ankle fractures in snowboarders.  It was a rare high-energy type injury prior to the sport of snowboarding.

Quadricep tendon ruptures are more common than patellar tendon ruptures.

Donapudi        5 Slide Follow Up

Treatment priorities for cholangitis:  Agressive fluid resuscitation (think sepsis fluids),  broad spectrum antibiotics, ERCP, and long term the patient will need cholecystectomy.  In patients who are not good surgical candidates acutely, IR can place a percutaneus drain to get source control of an infected gall bladder.   It is prudent to consult GI and General Surgery on all cholangitis patients.

Predictors of need for ERCP:  High bilirubin, thrombocytopenia, low albumin, and tachycardia.

Friend        5 Slide Follow Up

Vitamin K deficiency affects factors 2,5,7 ,9, 10, C, and S.   Vitamin K deficiency acts like a warfarin overdose.  It is rare in healthy adults. It can be caused by antibiotics, fat malabsorbtion, high doses of vitamins A and E.    Factor 7 is the factor affected most early and severely due to vitamin K deficiency or warfarin overdose.

Erbach    5 Slide Follow Up

Brain AVM's present with bleeding or seizures. 

AVM's are a high pressure malformation.&nbsp; When they rupture they have big bleeds.&nbsp;&nbsp; Cavernous Malformations are relatively low pressure malformations and present with seizures and smaller bleeds.

AVM's are a high pressure malformation.  When they rupture they have big bleeds.   Cavernous Malformations are relatively low pressure malformations and present with seizures and smaller bleeds.

Olfactory hallucinations are associated with temporal lobe seizures.

Denk       Safety Lecture   How to determine if a patient has fluid responsive septic shock

Fluid responsiveness is defined by increase in stroke volume of 10-15% with a 500ml fluid bolus.

There are new gadgets that can determine fluid responsiveness:  The EV-1000 can calculate stroke volume variation based on non-invasive finger probe readings.   The problem with this device is that patients have to be in sinus rhythm, not tachycardic, and have to be intubated to get accurate readings.

The other device is the Cheetah NiCom. 

There is no Tech fix or fool proof way to measure fluid responsiveness in septic patients. We need to continually re-evaluate our septic patients.  Give fluids mindfully and assess the patients' response to fluids.

Elise comment: From our Process Trial experience, most septic shock patients whether or not they have heart disease, CHF or ESRD can tolerate 30ml/kg of fluids. You need to frequently re-evaluate these patients.  But in general give 30ml/kg for septic shock. 

 

 

Conference Notes 5-24-2017

Putman/Ohl    Oral Boards

Case 1.  50 yo male with fatigue and low grade fever.  Patient has a petechial rash/skin bruising. Labs show anemia and low platelets. Peripheral smear shows shistocytes. Bun/Creatinine was elevated.   Patient has mental status change as well. Diagnosis is TTP.

Classic Pentad for TTP.&nbsp; However, the Triad of anemia, thrombocytopenia, and neurologic symptoms without fever and renal dysfunction is more common than the Pentad.

Classic Pentad for TTP.  However, the Triad of anemia, thrombocytopenia, and neurologic symptoms without fever and renal dysfunction is more common than the Pentad.

Treatment of TTP includes Plasma Exchange (PEX), andIV steroids.   TTP is due to ADAMTS13not cleaving VWF and platelets. Because of this ADAMTS13 dysfunction, the patient gets long VWF and marked platelet aggregation.   When considering TTP, check the level of ADAMTS13 activity.   Do not transfuse platelets unless there is life-threatening bleeding.  More platelets can cause larger platelet aggregates.  Get plasma exchange done as rapidly as possible.

Elise and Harwood comment: Avoid doing an LP and central line in non-compressible sites when the platelet count drops below 50,000.  

 

Case 2.  36 yo male presents with dental pain, fever, and brawny edema of the submandibular area.

*Ludwig's Angina is the diagnosis.  Treat with broad spectrum antibiotics and consult for ENT surgical evaluation.   Be prepared for difficult intubation.

Case 3.   35 yo female bit by her cat.  Treatment includes prophylactic antibiotics to cover pasturella multocida.  Augmentin is the usual choice for patients who are not PCN allergic. For PCN allergic patients, bactrim + Clindamycin or doxycycline + clindamycin or cipro + clindamycin are options.   Update tetanus status. Cat bites are low risk for rabies.  Check an xray for fracture or foreign body.

*Tip of the Cap:  Bristol Schmitz was nominated for an MVP Award*

Lovell/Williamson      Spiritual Wellness & Mindfulness

Resilience is foundational to wellness.   

One simple thing you can do to develop wellness is at the end of each day think of 3 good things that you experienced that day.

Spiritual Wellness includes: Nurturing your deliberate actions.  Having the ability to reset your attitude or thinking.  Finding purpose and meaning in your work.  Employing mindfulness, meditation, and or religion to reach spiritual wellness.

Mindfulness is a tool to improve our spiritual wellness.   Mindfulness is paying attention to the moment with purpose, and not passing judgment on the moment or the thoughts we have at that moment. This can include meditation or conscious breathing or just presence in the moment.  This is the opposite of multi-tasking. It is focused mono-tasking.  

Victor Frankl made the point that mindfulness helps us learn about the space between a stimulus we experience and our response to that stimulus. We can control that space and positively impact our behavior.

You can start to meditate by doing it 10 minutes a day for 10 days in a row.  "Head Space" is a website developed by a buddhist monk that can help you meditate.   Research has shown that Meditation develops your anterior cingulate cortex.

Tekwani       Study Guide  

WASH regimen for anal fissures: Warm water (sitz baths), Analgesia, Stool softeners, and High fiber.

With exposure to hepatitis B infection patient will develop anti-HBS and anti-HBC.   With vaccination, the patient will only have anti-HBS and not anti-HBC.  Editorial comment: For test purposes maybe remember C stands for Close Contact with source person. Kinda weak mnemonic but the best I could come up with.

Amebic abscess is usually single and has a rim of edematous tissue.&nbsp; Amebic abscess is treated with metronidazole.&nbsp; Patients with amebic liver abscess may have an associated right side pleural effusion.

Amebic abscess is usually single and has a rim of edematous tissue.  Amebic abscess is treated with metronidazole.  Patients with amebic liver abscess may have an associated right side pleural effusion.

Strangulated hernias will have abdominal and or systemic signs due to impaired blood supply to the affected bowel.  Patients will have fever, tachycardia, and peritonitis. Don't reduce suspected strangulated hernias in the ED.  You may perforate the bowel. Consult surgery and start antibiotics.   Hernias with no fever, no peritonitis, and no systemic symptoms can have an attempt at reduction in the ED. 

Cecal volvulus seen in young patients,&nbsp; usually has dilated bowel in left upper quadrant on test questions.&nbsp; Also look for dilated small bowel loops associated with cecal volvulus.&nbsp;&nbsp; Sigmoid volvulus seen in elder patients, usual…

Cecal volvulus seen in young patients,  usually has dilated bowel in left upper quadrant on test questions.  Also look for dilated small bowel loops associated with cecal volvulus.   Sigmoid volvulus seen in elder patients, usually has dilated bowel in right upper quadrant and has dilated large bowel most prominantly. 

Dawson        Pediatric Airway Emergencies

Failure to manage the airway is the leading cause of preventable deaths in children.

The Pediatric airway is narrowest at the subglottic cricoid ring. The pediatric larynx is more superior in the neck compared to adults. The infant tongue is larger than the adult tongue.

Dr. Dawson rarely uses an uncuffed ET tube. As a general rule cuffed ET tubes are preferred for most pediatric patients.

Dr. Dawson rarely uses an uncuffed ET tube. As a general rule cuffed ET tubes are preferred for most pediatric patients.

Lemierre's syndrome (or Lemierre's disease, also known as postanginal shock including sepsis and human necrobacillosis) refers to infectious thrombophlebitis of the internal jugular vein.[1] It most often develops as a complication of a bacterial sore throat infection in young, otherwise healthy adults. The thrombophlebitis is a serious condition and may lead to further systemic complications such as bacteria in the blood or septic emboli.

Lemierre's syndrome occurs most often when a bacterial (e.g., Fusobacterium necrophorum) throat infection progresses to the formation of a peritonsillar abscess. Deep in the abscess, anaerobic bacteria can flourish. When the abscess wall ruptures internally, the drainage carrying bacteria seeps through the soft tissue and infects the nearby structures. Spread of infection to the nearby internal jugular vein provides a gateway for the spread of bacteria through the bloodstream. The inflammation surrounding the vein and compression of the vein may lead to blood clot formation. Pieces of the potentially infected clot can break off and travel through the right heart into the lungs as emboli, blocking branches of the pulmonary artery that carry blood with little oxygen from the right side of the heart to the lungs.

Sepsis following a throat infection was described by Schottmuller in 1918.[2] However, it was André Lemierre, in 1936, who published a series of 20 cases where throat infections were followed by identified anaerobic sepsis, of whom 18 patients died.  Wikipedia reference.

 

 

 

Conference Notes 5-17-2017

Anderson/Traylor    Oral Boards

Case 1. 56 yo male with shortness of breath. HR 126, BP 88/59,  RR35.  On exam, patient has a diastolic murmur and rales bilaterally.   EKG shows LVH.  Echo shows signs of aortic root dilation and aortic regurgitation.  Diagnosis is aortic dissection with aortic valve failure.  CXR shows pulmonary edema.

Aortic regurgitation can occur due to aortic root dilatation or direct involvement of the aortic valve

Aortic regurgitation can occur due to aortic root dilatation or direct involvement of the aortic valve

 

Case 2. 36yo male with low back pain and bilat lower extremity pain.  Patient is tachycardic. He states he has an aching in his thighs.  Patient gives a history of ETOH and cocaine use the night prior. Urine showed large blood with minimal RBC's.  CK was 24,000.

Cocaine-induced rhabdomyolysis with secondary acute renal failure

By Karthikram Raghuram, MD, Department of Radiology, University of Alabama at Birmingham and Birmingham VA Medical Center, Birmingham, AL

The incidence of rhabdomyolysis in patients who use cocaine varies from 5% to 30% in published reports. It is unclear why cocaine causes rhabdomyolysis. Hypotheses include cocaine-induced vasospasm with resultant muscle ischemia, excessive energy demands placed on the sarcolemma, and direct toxic effects on myocytes. Seizures, agitation, trauma, and hyperpyrexia may also play a role. In general, the severity of the rhabdomyolysis parallels the severity of the cocaine intoxication; patients with very high CK levels tend to have the most severe complications from this disease. Intravenous cocaine use may be associated with a higher incidence of rhabdomyolysis-induced acute renal failure (ARF) compared with smoking cocaine.1

Patients with rhabdomyolysis classically present with complaints of muscle weakness, swelling, and pain. The myalgias may be focal or diffuse, depending on the underlying cause of the disease. The patient may also note dark- or tea-colored urine. However, a high clinical suspicion for rhabdomyolysis must be maintained in patients at risk because up to 50% of those with serologically proven rhabdomyolysis do not report myalgias or muscle weakness.1

 

Case 3. 45 yo male with bilat lower extremity numbness.  Vitals are normal. Numbness is localized to lateral thighs.  Patient is obese.   Neurologic exam shows diminished sensation on the lateral aspect of thighs bilat.  Remainder of motor and sensory exam is normal. Diagnosis is meralgia paresthetica.

Geraghty    Endovascular Treatment of Stroke

The typical stroke patient loses 2 million neurons per minute

Only 8% of stroke patients are eligible for TPA.  Also, TPA does not work well for larger clots.

Patients with life changing or potentially fatal strokes are candidates for endovascular management.   Minimum NIH stroke scale of 6 for considering endovascular therapy.

Stent thrombectomy after TPA resulted in earlier neurologic recovery and improved functional outcome at 3 months compared to TPA alone for patients with proximal clots.   NNT=3. 

Who gets endovascular therapy?   No bleed on plain CT.  CTA shows clot in proximal anterior circulation. Patiet received TPA within 4.5 hours and can get endovascular procedure within 6-12 hours.  New study just released today shows that even patients who wake up from sleep with stroke symptoms can benefit from endovascular therapy without receiving TPA.

If a patient wakes up with stroke symptoms and has an NIH score >6 send them for both plain CT and CTA.  If they have a large vessel occlusion they may be eligible for endovascular therapy without TPA.  Dr. Garaghty will consider patients for endovascular therapy based on imaging and the overall clinical picture not just strictly on time endpoints.  

We had a general discussion of how this new data will affect the ED approach to stroke.  We will need to start considering endovascular therapy for a much larger subset of patients than those that present with stroke symptoms within 4.5 hours. Because evolving evidence is suggesting that patients with proximal clots may benefit from endovascular therapy even if they are not TPA eligible based on time of onset.

Berkelhammer/Carlson     Acute Liver Failure

The INR is the best indicator of liver function/dysfunction.

Acetaminophen overdose is the most common cause of acute liver failure.  Acetaminophen causes centrilobular necrosis due to the concentration of cellular mechanisms to to detoxify acetaminophen in the centrilobular areas. 

Mechanism of acetaminophen detoxification in the centrilobular region of the liver. NAPQI is the toxic metabolite of acetaminophen. NAC/glutathione detoxifies NAPQI.

Mechanism of acetaminophen detoxification in the centrilobular region of the liver. NAPQI is the toxic metabolite of acetaminophen. NAC/glutathione detoxifies NAPQI.

The RM nomogram can only be used for acute single ingestions of acetaminophen.&nbsp; You don't need to adjust the nomogram for patient factors such as chronic or acute ETOH use, P450 inducing medications, or malnourishment.&nbsp; There is enough saf…

The RM nomogram can only be used for acute single ingestions of acetaminophen.  You don't need to adjust the nomogram for patient factors such as chronic or acute ETOH use, P450 inducing medications, or malnourishment.  There is enough safety built into the nomogram to cover all patients.

NAC prevents severe liver injury if given within 8 hours of ingestion.  There are benefits though even if NAC is given later. 

If you use the 21 hour IV protocol you need to verify there is no detectable acetaminophen in the blood and the LFT's are normalizing prior to stopping therapy. If not, then you need to treat beyond 21 hours.&nbsp; There have been some reported case…

If you use the 21 hour IV protocol you need to verify there is no detectable acetaminophen in the blood and the LFT's are normalizing prior to stopping therapy. If not, then you need to treat beyond 21 hours.  There have been some reported cases of liver failure when the patient was treated with IV NAC for 21 hours only and the acetaminophen level was not 0.

Dr. Berkelhammer only will allow a total of 2 grams of acetaminophen per day in patients with alchoholism or cirrhosis. In patients with cirrhosis he also avoids NSAID's to reduce the risk of GI bleeding and renal failure.   If patient needs more pain control he favors prescribing norco with higher doses of hydrocodone (norco 7.5 or norco 10)

Twanow   5 Slide Follow Up

Erythrodermic Psoriasis

Erythrodermic Psoriasis can look like TEN.&nbsp; Treatment of Erythrodermic Psoriasis includes IV fluids, systemic steroids, and local wound care.&nbsp; Care best provided in a burn unit.

Erythrodermic Psoriasis can look like TEN.  Treatment of Erythrodermic Psoriasis includes IV fluids, systemic steroids, and local wound care.  Care best provided in a burn unit.

 

Denk      Trauma Airway

To obtain an airway in this patient, you can cut the vertical wires with trauma shears or wire cutters to open the mouth enough to get a Video or Direct Laryngoscope in the mouth.&nbsp; If you don't have time to do that you can either attempt nasotr…

To obtain an airway in this patient, you can cut the vertical wires with trauma shears or wire cutters to open the mouth enough to get a Video or Direct Laryngoscope in the mouth.  If you don't have time to do that you can either attempt nasotracheal intubation or perform a cricothyrotomy.

Dr. Denk discussed 2 other difficult trauma airway situations.  She discussed a recent EmCrit Podcast "Having a Vomit SALAD"  which discusses using the yankaur suction to lead the laryngoscope blade into the supraglottic space to suction out blood or vomitus.  You can then move the suction catheter to the left side of the mouth and keep the tip in the upper portion of the esophogus.  This way the suction catheter continues to clear blood or vomitus from the airway while you are trying to intubate.

Einstein        Extremity Trauma

Every extremity injury requires an evaluation of vascular status, nerve function, soft tissue injury, and bony injury.

When evaluating for vascular injury in an extremity, examine for hard signs, soft signs, and get ABI's.

When evaluating for vascular injury in an extremity, examine for hard signs, soft signs, and get ABI's.

Conference Notes 5-10-2017

Munoz   M&M

Pay attention to vital signs in kids.  Pediatric patients can look OK despite significantly abnormal vital signs.  Vital sign abnormalities, especially tachycardia above age-appropriate values may be the only early clue to serious illness.

Early IV antibiotics and IV fluids are as critical in kids as in adults for treating sepsis.

Be cautious of pediatric patients who have a return visit to the ED for the same illness. They may warrant a more detailed evaluation.   The average emergency physician during their career will send home 44 patients who will die within 7 days.

Risk factors for death in discharged patients: abnormal vitals especially tachycardia, failure to recognize worsening of chronic illness, atypical presentations of disease, patients with psychiatric illness or substance abuse. If a patient has one or more of these risk factors, consider further evaluation or a period of observation prior to discharge.

Another interesting 2017 study discussing deaths in discharged medicare patients.&nbsp; Higher admission rates are associated with less deaths.&nbsp; ED discharge diagnoses such as altered mental status, dyspnea, and malaise, were the highest risk f…

Another interesting 2017 study discussing deaths in discharged medicare patients.  Higher admission rates are associated with less deaths.  ED discharge diagnoses such as altered mental status, dyspnea, and malaise, were the highest risk for death after discharge.

BMJ 2017; 356 doi: https://doi.org/10.1136/bmj.j239 (Published 01 February 2017) Cite this as: BMJ 2017;356:j239

Elise comment:  Once you make the determination that a patient is ill and needs time-critical interventions,  you need to physically look at the clock and set definite time constraints on your team to get things done.  If you don't hit those time marks you need to stop all other activity and re-focus the entire team on the patient you are concerned about.

Patients who are boarding in the ED for prolonged periods of time can have significant changes in their condition.  Try to at least lay eyes on these long term sign out patients at least once during your shift. 

Submassive PE is defined by RV dysfunction or elevated troponin

Massive PE is defined by hypotension <90 mm HG lasting more than 15 minutes or requiring inotropic support, shock, profound bradycardia, or cardiac arrest

*Management of submassive and massive PE

Katiyar     Toxicology

The intrinsic and extrinsic coagulation pathways converge at Factor 10

 Excerpt from DeSancho TM, Pastores SM The Liver and Coagulation:   The liver is the primary site of synthesis of most of the clotting factors and the proteins involved in the fibrinolytic system. These include all the vitamin K-dependent coagulation proteins (factors II, VII, IX, X, protein C, protein S and protein Z), as well as factor V, XIII, fibrinogen, antithrombin α2-PI and plasminogen. The notable exceptions are von Willebrand factor (VWF),tPA, thrombomodulin, TPFI and uPA. The VWF, tPA, throm-bomodulin and TFPI are synthesized in endothelial cells, while uPA is expressed by endothelial cells, macrophages, renal epi-thelial cells and some tumour cells [4].

Vitamin K, a fat-soluble vitamin, is required to achieve proper levels of procoagulant factors (II, VII, IX and X) and anticoagulant factors (proteins C, S and Z). These factors require vitamin K as a cofactor for post-ribosomal modification to render them physiologically active.

Warfarin blocks Vitamin K reductase and affects the factors 2,7,9,10, and proteins C,S, Z

Warfarin side effects include blue toe syndrome, skin necrosis, hair loss, and urticaria in addition to bleeding problems.

*Warfarin Blue Toe Syndrome is a rare complication of warfarin.  If is thought to be due to cholesterol emboli released from bleeding within an arterial plaque.  If you stop the warfarin, the toes will remain blue but the pain will improve.  After stopping warfarin, changethe patient to a 10a inhibitor for anticoagulation. Consider other sources of thromboembolism such as an aortic aneurysm.

 

*Foods that have a high level of vitamin K.  Abhi discussed a case in which an elderly patient was not eating any vegetables because she lost her dentures and had an increasing INR because her dietary vitamin K intake had significantly decreased.

If a child ingests rat poison (super-warfarin) you don't need baseline labs on initial presentation.  Kids need coagulation labs 24-48 hours after ingestion. If at that time INR is prolonged or child has bruising/bleeding you would initiate vitamin K therapy.  Life threatening hemorrhage in this situation may require FEIBA for management.

 

*Factor 10a and 11a Inhibitors

Reversal of 10a and 11a inhibitors is accomplished by administering FEIBA.

Altman/Katiyar/Williamson        Medico-Legal Small Group Workshop

Chan/DeWeert     Oral Boards

Case 1.  67 yo female with abdominal pain.  Patient has history of Atrial fibrillation. She is not on warfarin or 10a inhibitor.  On exam pt has diffuse abdominal tenderness. Stool is heme positive. Lactate is 4.6.  Diagnosis is mesenteric ischemia due to an embolism to the SMA confirmed.   Diagnosis made by CTA showing occluded SMA.  Treatment is IV fluids, IV morphine, IV heparin, and IV antibiotics.  Emergent consult to Vascular Surgery and IR.

Which consultant provides definitive treatment (Vascular surgery, GI, or IR) will have to be determined on a case by case basis.  The clinical situation will determine which consultant can bring the most applicable skill set to the patient's care.  Some of the residents discussed cases where the GI consultant scoped the patient prior to surgery to determine the extent of necrosis. Depending on the type and extent of the vessel occlusion and how much the bowel is affected will determine whether Vascular Surgery or IR treats the patient.

 

Case 2. 33yo male with right ankle and foot pain after a fall.  Xrays show a calcaneus fracture.  Examine the spine and entire lower extremity for associated injuries in a patients with a calcaneus fracture.  Initial management is splinting, elevation, non-weight bearing with crutches or walker, pain management and orthopedic consultation.  Many patients with calcaneal fractures can be discharge home with outpatient follow up.  However, there are risks of compartment syndrome of the foot, fracture blisters, and associated injuries of the lower extremities, spine, pelvis and abdomen in patients with calcaneal fractures. .

Case 3.  54yo male with epigastric pain.  He is tachycardic.  Finger stick blood sugar is >600.  Labs are consistent with DKA (anion gap, metabolic acidosis, ketonuria). Patient has an elevated troponin but no STEMI on EKG.  Diagnosis is DKA with NSTEMI.  Treat with IV fluids, IV insulin, potassium, po asa, and IV heparin.

Dave Barounis taught me to focus on the anion gap and urinary ketones.&nbsp; If both are present in the setting of hyperglycemia, you have DKA.

Dave Barounis taught me to focus on the anion gap and urinary ketones.  If both are present in the setting of hyperglycemia, you have DKA.

Okubanjo      Safety Lecture

I missed this excellent lecture

Chiefs    Ortho Cases

I missed this excellent lecture

Conference Notes 4-19-2017

Thanks to Elise Hart for her help with the Conference Notes for the Ophthomology Lecture by Drs. Farooq and Shah this week. 

Okubanjo/Ryan     Oral Boards

Case 1.   19yo male passes out at a movie theater. No seizure activity identified.  EKG shows a Brugada pattern.

Brugada Pattern in Leads V1-2

Brugada Pattern in Leads V1-2

Patient required admission to cardiolgy service for AICD placement.

Case 2. 41 yo male with right leg pain following a bad ankle injury.  Vitals are normal.  Xrays show maisoneuve fracture.

Maisoneuve fracture pattern has a medial maleolar fracture or tear of deltoid ligament with associated proximal fibular fracture and tear of the syndesmosis between the tibia and fibula.&nbsp;&nbsp; On exam, palpate both the ankle and the proximal f…

Maisoneuve fracture pattern has a medial maleolar fracture or tear of deltoid ligament with associated proximal fibular fracture and tear of the syndesmosis between the tibia and fibula.   On exam, palpate both the ankle and the proximal fibula to identify this fracture pattern.  Treatment is splinting followed by ORIF.

 

Case 3.  45 yo male with weakness and vomiting. Patient has tachycardia with thready pulses. Patient has history of daily ETOH use and recently has had vomiting and cannot keep down any food/fluids.  Diagnosis is AKA. 

Lovell      Town Hall Meeting

Traylor     Stroke Outcomes in Patients over 80yo Receiving TPA at ACMC

Logan gave his upcoming ICEP presentation.  I did not want give away the info prior to his presentation.  You will have to attend ICEP to get the outcome info. 

Okubanjo      Healthcare Disparities

Oyin gave her upcoming CORD presentation describing the Healthcare Disparities Curriculum she created.

Einstein     Wilderness Medicine

Noah gave his upcoming CORD presentation describing the Wilderness Medicine Curriculum he developed.  

Pastore     5 Slide F/U

There are 2 conditions in LVAD patients that require immediate notification of LVAD team.

#1 Pump failure which will be indicated by screaming LVAD alarms

#2 Pump thrombosis indicated by dark or brown urine.  Patients will have an elevated LDH and signs of hemolysis on CBC.  Patients may have new heart failure symptoms.  The incidence of pump thrombosis has increased recently due to a trend toward lower anti-coagulation INR targets for LVAD patients. 

Treatment of pump thrombosis is heparin drip and IV bicarb drip. Patient may go to OR for LVAD exchange or ECMO.  TPA can also be used. Get the LVAD team involved in the patient's care as soon as possible.

Ashley recommended asking all LVAD patients what their urine looks like to screen for pump thrombosis/hemolysis.  Also get an LDH on all LVAD patients to screen for pump thrombosis/hemolysis.  Compare the LDH to prior levels. 

Tran       5 Slide F/U

Treat delerium tremens with IV Ativan and IV phenobarbital.

Etoh withdrawal seizures don't typically have prolonged post-ictal periods.

The later a patient starts having withdrawal symptoms following cessation of ETOH, the worse the prognosis/severity of withdrawal.

Phenobarbital works at the GABA receptor and also lowers glutamate in the CNS.

Dexmedetomidine is a newer sedation medication that can be very effective in ETOH withdrawal. The downside is that it is very expensive.

Kennedy comment: When giving patients big time bnezo's and phenobarb keep them on an end-tidal CO2 monitor to be alert for potential apnea. 

Hart/Regan    Ortho Updates

Rotator Cuff Tears can be diagnosed with the following exams:

Treat with sling and Ortho Management.  Some patients will require surgery

Farooq/Shah   Visiting OphthalmologyConsultants from U of C     Eye Emergencies

Ruptured globes - mechanism important. look for abnormality to pupil as one clue

 

Usually no rush to get FBs out unless contributuing to increased IOP (very rare) or if wood/vegetable matter (very inflammatory) - o/w FBs unlikely to get infected.

 

lateral canthotomy/cantholysis:

1) inject lido WITH epi to help w/ hemostasis/can also clamp down with hemostat

2) cut skin with 15 blade first (scissors often too dull, though )

3) Get scissor between globe and cathus, keeping dull side of scissor against globe. Cut posterior (lateral canthotomy) first (note this part doesn't do much for IOP), then inferior limb (cantholysis)

 

Pearls: 

- If you can pry open lid and their EOMs are OK they're unlikely to need lateral canthotomy

- You can strum with scissors to see if you feel the cord of the tendon to know if you got it

- Don't worry too much about messing up the lid/skin - oculoplastics can always fix that later

- If pressure <22 they're probably fine

- On call Ophtho should come in if this is being done - OK to start without them, but their job is to ensure it was done adequately

- If being done for compressive optic neuropathy should also give steroids (recommended dose: 500 mg? solumedrol).  Exception: Traumatic Optic neuropathy (posterior orbit sphenoid fx w/ effect on vision but not EOMs and not a lot of external swelling --> this involves direct trauma to optic nerve, akin to lack of efficacy of in spinal cord injury)

If there is a severe eyelid laceration exposing the globe, place antibiotic ophthalmic ointment as soon as possible to protect the globe.  Talk to plastics or ophthomology to emergently approximate the lacerated lid to get coverage over the globe.   If you as an emergency physician need to repair a gaping eyelid laceration that exposes the globe use 5.0 or 6.0 vicryl or 5.0 or 6.0 fast absorbing plain gut.  Keep the sutures in the tarsal plane external to the mucosal lining as much as possible.

Injuries to the medial canthus area that damage the canaliculi system need to have a plan to repair the canulica within 3 days.  Beyond 3 days there is significant scarring and it limits ophtho's ability to repair the problem.

Orbital floor fractures are almost never an emergency in adults unless there is an associated serious eye injury.   Pediatric orbital floor fractures need to be addressed on an emergent basis because pediatric fractures under age 16 can entrap and strangulate the inferior rectus muscle causing life long diplopia.  If a pediatric patient has inability to elevate the eye in the injured orbit they need emergent surgery.

Chemical Burns Check the ph in 4 quadrants of the eye.  Check visual acuity, pupils, and eye pressure.  Alkali burns cause more damage than acid burns because alkali causes liquifaction necrosis.  Acute conjunctival irritation is actually a good sign, better than a whitish appearing cornea.   Copious irrigation of the eye is the key management to chemical burns. Irrigate until the ph is 7 in all 4 quadrants.  It may take 10-16L of NS to get ph down to 7.  Chemical burns can cause severely elevated intraoccular pressure.   Adjunctive therapy for chemical burns includes topical steroid, topical antibiotic, and cycloplegic drop.  Other therapies include vitamin C, doxycycline, and intraoccular pressure lowering drops.

Central Retinal Artery Occlusion is characterized by painless unilateral vision loss.  Patient should be admitted or stroke work up.  Neurology should be consulted.  There is no proven therapy for this disease.  Occular massage may be helpful.  Long term visual prognosis is poor.  Rare diagnosis in children.  Can be associated with malignacies or patent foramen ovale in children. 

Globe rupture signs: 360 degree conjunctival hemorrhage, 8 ball hyphema, flat anterior chamber, irregular pupil.  Get a CT scan to evaluate the globe.  For adults, Avelox is the optimal antibiotic for globe rupture because of it's abilty to attain high levels in the globe.  Levaquin is the second choice.   In kids, discuss antibiotics with ophthalmology.  Some pediatric specialists may advocate for a single dose of avelox in kids as well.

Acute Glaucoma treatment

Acute glaucoma management

Don't send topical ophthalmic anesthetics home with patients with eye pain.  It can cause complete vision loss in the affected eye.    For pain relief for corneal abrasions use topical antibiotic ointment and an oral analgesic.

Conference Notes 4-12-2017

Hart/Regan      STEMI Conference

Case 1.   Patient had environmental hyperthermia associated with lateral STEMI changes.  Decision between Emergency Physicians and Cardiology agreed that rapid cooling was indicated rather than emergent cardiac catheterization.   EKG changes resolved with rapid cooling.   Patient had markedly elevated CK due to Rhabdomyolysis.   Rhabdo was treated with IV fluids and eventually dialysis.  

There is a paper describing EKG changes in patients participating in the Haaj in Mecca who had hyperthermia.  Common EKG abnormalities seen were sinus tachycardia, QT prolongation, non-specific ST changes, and focal ST changes.  There are other case reports of STEMI appearing EKG changes in patients with hyperthermia.  Typically patients had resolution of EKG changes with rapid cooling.  Many had clean caths.   The EKG findings of hyperthermia are thought to be due to physiologic stress, dehydration, electrolyte abnormalities.

Harwood comment:  The treatment is to cool these patients not take them to the cath lab.  The Cardiology faculty present agreed.  They said treat the patient, not the EKG.  Cooling is the main therapy in this situation.

Case 2.  Elderly patient with prior CAD, stents, HTN presents with syncope.  Patient's initial EKG showed clear-cut STEMI changes.  Patient had history of placement of coronary stents for a prior STEMI about a week prior to this presentation. Cardiac Cath at this second presentation showed thrombosis at proximal portion of recently placed stent.

There is no clear-cut time frame for STEMI changes to resolve following MI.  If ST elevation is identified beyond 1 week post-STEMI that is probably an acute abnormality or an aneurysm or pericarditis.

Stent thrombosis is uncommon with about a1-2% frequency.  Stent thrombosis sually presents with STEMI or death.  Patients not taking anti-platelet therapy are at higher risk for stent thrombosis.  Bare metal stents most commonly occlude in the first 48 hours.  Drug eluting stents most commonly re-occlude within 30 days.  STEMI's secondary to stent thrombosis have worse outcomes than STEMI's caused by plaque rupture.  Cardiac Cath is very time-sensitive in these patients.

Cardiology comment: Most of the time stent thrombosis is due to either medication non-compliance or mechanical/placement issues related to the stent.

Cardiology comment: It is very important in patients with prior CAD and/or stents to compare their ED EKG with a prior EKG.  The patient may have aneurysmal changes or LVH changes that can look like acute STEMI.  If you have an old EKG showing similar changes it can help clarify the situation.

Case 3. Elderly patient presents with dyspnea. EKG showed a paced rhythm with anterior concordant ST depression.

 

Sgarbossa criteria for identifying STEMI in patients with LBBB and Paced rhythm.&nbsp; The main thing to look for is concordant ST depression or elevation.

Sgarbossa criteria for identifying STEMI in patients with LBBB and Paced rhythm.  The main thing to look for is concordant ST depression or elevation.

 

 

Smith Modification of Sgarbossa criteria: If you identify concordant ST elevation or depression, that is significant.  If you have discordant ST change more than 25% of the main deflection of the QRS, that is also considered significant.   This last criteria replaces the 5mm discordant criteria from Sgarbossa.

Cardiology comment:  The decision to take patients to the cath lab with paced rhythms is difficult.  The decision has to be made on a case by case basis using Smith modified Sgarbossa criteria and looking at the overall clinical picture.

KennedyM&M

I only noted the take home points to avoid divulging the specific aspects of these cases.

When pacing, you have to set the asynchronous rate above the patient's native rate or you can get a pacer spike landing on a QRS complex.  This can cause V-tach, torsades, or V-fib.  If you are technically adept with the pacer device, placing the pacer in VVI mode also avoids this R on T phenomenon.

Harwood and Girzadas comment: There is a value to keeping things simple when placing a transvenous pacer.  The "Emergency" button on the pacer give you asynchronous pacing at the rate of 80 which does the job in the vast majority of patients who have a very low native rate.  Harwood made the point that once the patient is more stable you can then adjust the pacer box to the VVI mode. 

Carlson comment: Check the pacer battery while you are prepping to place the line.  It is not uncommon that the pacer battery may need replacement. You don't want to find out the battery is not working right when you need the device to pace.

Check vital signs prior to discharging patients. Document any re-assesments thatyou perform on patients.

Consider getting an EKG on patients that are persistently tachycardic with no good clear diagnosis.  Be on the lookoutfor myocarditis.

Tekwani comment: If you get a gut feeling about a patient in signout, go to the bedside and carefully re-evaluate the patient.  Don't ignore that gut feeling.

Lovell comment: Be cautious about chalking up symptoms and signs to anxiety.  Most of the time the diagnosis is actually anxiety but rarely it can be an occult pathology.

Sedation and positive pressure intubation can markedly decrease cardiac output and increase the chance of cardiac arrest.  Be sure to resuscitate the patient prior to intubation. Have pressors ready to go or even started prior to starting intubation.

Williamson comment: Take some time prior to every acute resuscitation to speak with the family and appraise them of the gravity of the situation.  It should be in your pre-resuscitation check list.

Menon/EinsteinOral Boards

Case 1.  Botulism secondary to IV drug use.  Treat botulism with anti-toxin.  You can obtain anti-toxin from CDC.  The Illinois poison control also has access to anti-toxin

*Botulism

Carlson comment: Heroin is the #1 cause of wound botulism.

Case 2.  Patient with recurring fever following trip to Africa.  Malaria was suspected based on CBC showing anemia and schistocytes.  Thick and thin smears confirmed malaria. 

Elise comment: You need to know this diagnosis for the boards.

*Thick and Thin smears for Malaria

Case 3.  Flexor tenosynovitis Treat pain, update tetanus, give IV antibiotics and consult hand surgery. Admit patient for further management and surgery.

*Knavel's 4 signs of Tenosynovitis

Destefani         Safety with Pediatric IV fluid Administration in DKA

IV fluid administration in pediatric patients with DKA needs to be done with caution to avoid cerebral edema. However, research illuminating how exactly to give fluids in Peds DKAis lacking.  Cerebral edema in DKA is rare and relatively unpredictable.

Do not bolus NS in DKA patients unless the patient there are signs of shock. 

PECARN group is currently doing a study looking at various IV fluid strategies in Pediatric DKA patients.

Any change in mental status in a pediatric DKA patient should raise your suspicion for cerebral edema.  Also be alert for headache, incontinence, and vomiting.

*Diagnostic, Major and Minor Criteria for Cerebral Edema

Small studies show that central lines in pediatric patients with DKA may be more at risk for DVT.  If you can't get peripheral IV access go to an IO line first.  Favored locations for pediatric IO lines are proximal tibia, distal tibia, and distal femur. Avoid placing IO lines above the femur in skeletally developing kids. 

Harwood comment:  Most DKA patients are not in shock and don't require rapid fluid boluses or large fluid resuscitation.  Give them fluid and insulin per protocol only and they will gradually improve.

Garrett-Hauser     Ethics     Reporting Medical Errors

We discussed a case scenario regarding end of life decisions.

The act of withdrawing care is not furthering a patient's death.  It is returning the patient to their original disease state.   However when you withdraw care, you do need to fill out a "Goldenrod Form" justifying the withdrawl of care.  Two attendings need to co-sign the Goldenrod Form.

A patient had unintended imaging.  Patient was informed of imaging and radiation exposure. A discussion with billing office was had to avoid the patient receiving a bill for an unintended test.

If a patient has an adverse outcome from a medical error, the appropriate approach is to discuss the error, the cause, and the expected outcome with the patient and family.   Families want to know how this problem will be prevented going forward.    It is good to have Chaplain support with these conversations.

All these cases highlighted the importance of error disclosure AND the challenges we all have with informing patients and families about errors. 

 

Conference Notes 4-5-2017

Hart & Regan     Wound Management

Strategies to lessen the pain of injection of local anesthetic: small gauge needle, slow injection, inject through wound margin (not intact skin), buffer with bicarb, warm to body temp, and pre-treat with topical anesthetic such as LET.   Harwood comment: Use an Insulin syringe initially. It has a 31 gauge needle. Then use the longer 27guage needle to anesthetize more deeply.

Pasturella strains from cat bites are felt to be more virulent than pasturella strains from dog bites.  Additionally, cats have smaller, sharper teeth that can puncture more deeply further increasing the risk of infection.

Care of Amputated digit: Wrap the digit in saline soaked gauze, place the wrapped digit in a plastic bag, place the bagged/wrapped digit on ice.   Finally, keep the digit with the patient on the patient cart so the digit does not get lost.

Fight bite wounds are high risk for infection.  Examine all fight bite wounds through full ROM of the MCP joint to check for tendon injury.  Give Augmentin or Unasyn as initial antibiotic if patient is not penicillin allergic. Consult with Hand Surgery.  If there is evidence of extensor tendon injury, joint penetration, infection, or fracture, the patient should be considered for admission for washout.

To release skin entrapment in a zipper, you need to get sturdy wire cutters or a bolt cutter to cut the median bar of the zipper.  Dennis Ryan comment: This can be a tough procedure to get the median bar cut.

For lacerations around the eye, emergency physicians should probably not be repairing lacerations around the medial canthus, involving the lid margin, or if you see exposed fat.  If you identify any of these, you needto consult ophthomology.

When can packing after I and D be removed?  When purulence has stopped draining.

Wound VAC's work by decreasing edema, improving vacular and lymphatic flow, decreasing bacterial density and increasing granulation tissue.

Ohl/Yappo   CPC Presentations

I did not want to divulge the cases because they will be presented at CORD this month.

Lorenz    5 Slide Follow UP

Patient presents with dizziness.  EKG shows polymorphic ventricular tachycardia (Torsades)

The patient was alert and talking only because he had an LVAD.   Patient was initially given IV magnesium without effect.   Patient was next defibrillated into sinus rhythm.   In hospital patient was treated with amiodarone and AICD.

Data shows that 2 year survival with an LVAD is about 23%.  LVAD's can give patients a longer life. On the other hand, patientsmay experience challenges and complications from an LVAD.   This risk/benefit balance was discussed.

Harwood comment: If you see a patient with torsades who does not have an LVAD you need to defibrillate and also start magnesium and an anti-arrhythmic.  These patients will continue to revert to torsades after defibrillation until you get therapuetic levels of an anti-arrhythmic.

Wing    5 Slide Follow Up

Myasthenic crisis can be precipitated by infection, pregnancy, and medications (aminoglycosides, flouroquinolones, beta blockers, and magnesium).

Evaluate the patient with a measurement of Vital Capacity and NIF.  Normal VC should be around 3L (bad is around 1.5L).  Normal NIF should be around -60(bad is around -20).   If the NIF is -20 neuro may use plasma exchange.   Harwood would intubate if NIF is worse than -20.

Treatment is prednisone, IVIG, and plasma exchange.   Intubation for respiratory support.

Faculty comments: Respiratory therapy can measure both VC and NIF in the ED.

Snip20170405_8.png

Nejak    STEMI Equivalents

Dan's recommended steps to evaluating an EKG: Get an old EKG to compare.  Look at AVL for isolated depression.  Check for loss of precordial T wave balance.  Look for LMCA ischemia, Check for Wellen's and DeWinters signs. Check for reciprocal changes.  Check for hyper-acute T waves.

 

Loss of precordial T wave balance

 

 

LMCA Occlusion shows diffuse ST depression and ST elevation in AVR and V1

 

DeWinter's ST depression in V2-5 with rocket shaped T waves and Subtle AVR ST elevation.  This is an anterior STEMI equivilent and indicates LAD occlusion.

 

Wellen's is a sign of critical LAD stenosis.  These patients should have Cardiology evaluation.

DeWeert, Stanek,  Traylor      Abdominal and Pelvis Trauma

There was a ton of information in this excellent lecture that I could not encapsulate into these notes.  I put in a few key points made in the lecture.

Every 10 minute delay to give blood in trauma patients has an increased odds ratio of 1.27 for mortality.

Give TXA within 3 hours of initial trauma.  It is safe and number needed to treat is somewhere between 7-67 to decrease mortality.  Beyond 3 hours after injury, TXA increases mortality for unclear reasons.

Image shows the effect of a pelvic binder to close down the potential space in the pelvis.   Have a low threshold to place a pelvic binder early in the course of managing a multiply injured trauma patient.  There was consensus that the old strategy of "rocking the pelvis" to identify pelvic fracture on physical exam risks increasing pelvic bleeding.  In the multiply injured trauma patient, bind the pelvis early on in the resuscitation, don't rock the pelvis,  and get a pelvic x-ray to identify fracture.

Seat belt sign is associated with increased risk of intra-abdominal injury and Chance Fracture of the lumbar spine.

Algorithm implementing FAST exam in blunt abdominal trauma

 

Algorithm for Blunt GU Trauma

Algorithm for Blunt GU Trauma

There was a ton of information in this lecture that I could not encapsulate into these notes. 

Conference Notes 3-22-2017S

Stanek/Walchuk      Oral Boards

Case 1.   Patient with history of schizophrenia on multiple antipsychotic medications (Latuda, Lithium, Cogentin) presents with altered mental status. Patient has history of ataxia.  Patient has clonus on neurologic exam.  Lithium level was elevated.  Management included contacting poison control and initiating dialysis.  Any lithium level over 3.5 is considered severe.  Any change in mental status or significant neuro symptoms indicate dialysis.

Main point of discussion was to also consider serotonin syndrome and neuroleptic malignant syndrome in the polydrug psychiatric patient.

Case 2. 86yo female presents with 4 days of dyspnea. Patient has history of rheumatic heart disease and a mitral valve problem.   Lung exam reveals crackles. CXR shows cardiomegaly and congestive changes.  BNP is markedly elevated.  Patient's echo shows incompetent mitral valve.  Patient is in congestive heart failure due to severe mitral valve regurgitation.  Patient has valve rupture.  Treatment is afterload reduction with nipride followed by CV surgery consult.

Case 3.  7yo child suffered bite on finger from rat.  Treatment is copious irrigation of wound. Give antibiotics. No suturing or dermabonding. Tetanus prophylaxis is not indicated for rat bites.  People are at risk of Rabies from bites by Bats, Foxes, Raccoons, skunks, cattle, and horses.  No rabies risk from rats, squirrels, hamsters, guinea pigs.  The only lagamorph that can transmit rabies is groundhogs.  However, you can get rat bite fever which has a 13% mortality.   You can prevent rat bite fever with penicllin or augmentin at the initial treatment of wound.  Treat with penicillin, unasyn,or ceftriaxone.

Carlson     Oral Board Day Debrief

1. Unstable Afib requires cardioversion.  Cardioversion requires procedural sedation and a pre-sedation assessment. You also need to get informed consent, and do a Time Out.  Choose a sedative with minimal hemodynamic effects.

Cardioversion has a 90% success rate for unstable A-fib.  100J biphasic is the most commonly cited dosage for normal weight patient.  For obese patients start at 200J.  Anterior/Poster electrode pad placement seems to have a higher success rate.

Components of Pre-Sedation Assessment: Mallampati score, ASA Class, Prior Complications, Allergies, PO intake.

2. Consider abuse in the setting of an infant with altered mental status.  Check a blood sugar. Do a thorough physical exam to look for injuries.  Take custody of child.  Report the case to DCFS.

Infants are prone to head injury for shaken baby syndrome due to large heavy head compared to overall body size and large amount of water in head.   On exam look for bulging fontanelles and retinal hemorrhages, and inappropriate bruising. CT will show subdural or subarachnoid hemorrhage. Get a skeletal survey to look for prior fractures.

3. Patients with sickle cell disease are at increased risk for cholecystitis. They are also at increased risk for meningitis and infection from any encapsulated bacteria due to funtional asplenia.

4. Treat salicylate toxicity with alkalinization of serum/urine (goal: urine ph of 7-7.5) and hemodialysis. You have to replace potassium to effectively alkalinize the patient.  Method of alkalinization is initial 2 amps of bicarb bolus followed by bicarb drip of 3 amps in D5W and run at 4 hour rate.   This is an easy diagnosis to miss to elderly patients with chronic toxicity. Many OTC products have salicylate as a component. Goodies is an example.  Look for metabolic acidosis and respiratory alkalosis on the ABG.

5.  For neonates unstable with suspected congenital heart disease, give prostaglandin infusion. These kids will have mottling or cyanosis. Be prepared for apnea as a side effect of prostaglandin.  Prostaglandins can also have the side effect of fever and hypotension which can mimic sepsis.   If you have to transfer a neonate who is receiving prostaglandin, intubate prior to transfer. Lovell and Harwood felt that a septic work up in an unstable neonate may be potentially risky if you attempt to perform an LP.  Andrea felt you should still give antibiotics even if you have concerns about doing the LP.  

6. Be alert for testicular torsion in kids with lower abdominal pain.  Pediatric patients frequently will not say their testicle hurts.  Consult GU, Attempt detorsion. Get an ultrasound and get them to surgery.  Two peaks of incidence: First year of life and in adolescence.  If you can detorse or get to surgery in less than 6 hours there is a 90% salvage rate. 

7. Treat CO poisoning with 100% FIO2 with NRB and transfer for HBO. 

8.  Consider pericardial tamponade in patients with dyspnea and or hypotension/tachycardia. Electrical alternans is a sign of pericardial effusion. 

Electrical Alternans

Electrical Alternans

 

Echo is key to the diagnosis of pericardial tamponade.  Treat with pericardiocentesis and consultation for pericardial window. 

                                              Pecha Kucha

Alexander           Lipids to the Rescue!

Lipid emulsion has a protective effect from local anesthetic toxicity.  The best studied toxicity is from bupivicaine.

Lipid emulsion can be used also for toxicity due to tricyclics (amitriptylene, buproprion), beta-blockers and calcium channel blockers.

Hart    Vocal Cord Dysfunction

Predominantly seen in Caucasian females.  Can be confused with asthma, upper airway problems and panic attacks. Probably is due to combiniation of physiologic and psychiatric etiolgies. Standard dyspnea work up will be negative.  Flexible laryngoscopy will show paradoxical closure of the cords with inspiration.  The cords will have an opening at the base.

 Can treat with dissociative doses (0.5mg/kg) of ketamine.

Einstein   TEE for Cardiac Arrest

TEE gives you a better view of heart. Less frequent pulse checks.  It stays out of your way during resuscitation.  More sensitive test for cardiac contractility.  EM case reports and studies show anecdotal unexpected saves and improvement of CPR quality and diagnoses of dissections.  The probes are expensive.

Erbach   Cyanotic Congenital Heart Disease in the ED

If a child with a shunt has no murmur that indicates a big problem. Contact Peds CV surgery right away.

Use O2 to get patient back to their usual baseline, not higher than that.

Patients with a single ventricle have a higher risk of stroke.

Consider endocarditis in cardiac kids with fever.  Get blood cultures.

When consulting CV and Cards, know the patient's last procedure and when it occurred.

Donepudi       U/S in the Acute Management of Elevated ICP

Normal ICP is under 20mm Hg

Monroe-Kellie Doctrine

Monroe-Kellie Doctrine

You can use a high frequency probe to measure to optic nerve sheath diameter.  Use alot of ultrasound gel on a closed eyelid.  You can use tegaderm to keep the eyelid closed.  Measure 3 mm from globe.  Any diameter greater than 6 mm is abnormal.

Kennedy    Status Asthmaticus

Treat with aggressive bronchodilators, 10-20mg nebs.  Give 2grams of IV magnesium over 20 minutes.  IV steroids. Bipap using IPAP as the key to relieving work of breathing . Terbutaline 0.25mg SubQ. Epinepherine titrated as a drip.  Ketamine can be used for sedation for bipap or induction for intubation.  Intubation requires careful management to avoid air trapping and barotrauma.   Work to keep a limited plateau pressure.  Use a high I to E ratio.  Use smaller tidal volumes and lower respiratory rates.   Heliox can be used to improved air flow.  Heliox can't be used in hypoxic patients due to low FIO2 with Heliox.  Last ditch strategies are general anesthesia and ECMO/ECCOR (simplified ECMO).

Carlson    Toxicology

QT prolongation due to TCA can be treated with IV Bicarb and if that is ineffective, IV Lidocaine drip can be effective.

Physostigmine can be used in pure anticholinergic toxidromes that have coma, severe agitation, or intractable seizures.  Don't use empirically in patients with undifferentiated coma/agitation or polysubstance overdose.  Contraindications are TCA overdose, QRS wide, AV block, bronchospasm, bowel or bladder obstruction.   Andrea feels physostigmine is most indicated in pure anticholinergic overdose in pediatric patients. 

There was some difference of opinion between Harwood and Girzadas about the use of physostigmine.  Harwood felt he would use it in kids and teenagers with pure anticholinergic overdose and coma or severe agitation. It could potentially avoid intubation or complication of agitation. Girzadas was concerned about side effects of physostigmine particularly risk of bronchospasm and would do supportive care only (benzos for agitation, airway protection for coma) and avoid physostigmine. 

The common board question about an anticholinergic plant is Jimson weed.  Deadly nightshade, climbing nightshade, and Mandrake are other botanical anticholinergics. 

Treatment of TCA overdose includes sodium bicarb to a serum ph of 7.5.  Serum alkalinization is the goal not urinary alkalinization.  You are not trying to trap ions in urine. You are trying to keep the TCA out of the CNS.   Give bicarb for wide QRS, acidosis, hypotension, ventricular arrythmia.

TCA EKG from Life in the Fast Lane

Ohl    Safety Lecture

Sean discussed the content and organization of our ENT equipment.

Quick note on pancreatitis management: We need to treat pancreatitis with IV dilaudid for pain and somewhere around 350ml/hour of LR for the first 12 hours. The message from our GI consultants is to be more aggressive with our IV hydration for pancreatitis patients.